ДПР- домашние практические работы

Садыкова Рамзия Нурзадаевна

Практические работы для студентов ГАПОУ "Казанского строительного колледжа"

Скачать:


Предварительный просмотр:

МИНИСТЕРСТВО ОБРАЗОВАНИЯ И НАУКИ РФ.
Федеральное агентство по образованию
Государственное образовательное учреждение профессионального образования
КАЗАНСКИЙСТРОИТЕЛЬНЫЙ КОЛЛДЕЖ.


ДПР-ДОМАШНИЕ ПРАКТИЧЕСКИЕ РАБОТЫ

I КУРС


Тема
«Развитие понятия о числе»

Составитель преподаватель
математики Садыкова Р.Н.

КАЗАНЬ

ДПР Т1.1

Обучающая часть работы.

Абсолютная погрешность приближенного значения-это модуль разности точного значения и приближенного значения. Абсолютную погрешность можно применять для сравнения точности приближений одинаковых величин, а если мы собираемся сравнивать точности приближения разных величин, тогда одной абсолютной погрешности недостаточно.

Пример 1. На предприятии 1284 рабочих и служащих. При округлении этого до 1300 абсолютная погрешность составляет 1300-1284=16. При округлении до 1280 абсолютная погрешность составляет 1284-1280.

Относительная погрешность- это отношение абсолютной погрешности приближенного числа к самому этому числу.

Число ε называется границей относительной погрешности.

Границей относительной погрешности а приближенного значения а называется отношение границы абсолютной погрешности ∆а к модулю числа

Чем меньше относительная погрешность, тем выше качество измерений или вычислений. Относительная погрешность – величина безразмерная, что позволяет сравнивать качество измерений величин разной размерности

Пример 2.В школе 197 учащихся. Округляем это число до 200. Абсолютная погрешность составляет 200-197=3. Относительная погрешность равна  или, округленно,

 САМОСТОЯТЕЛЬНАЯ ЧАСТЬ РАБОТЫ

№1. Найдите абсолютную погрешность числа чисел: 5,26317, 2,02698 3,26593.

№2. Найдите относительную погрешность числа: 2,9658; 3,05698; 1,69583.

№3. Длина листа бумаги формата А4 равна (29.7±0,1) см. Найдите абсолютную и относительную погрешность.

№4. Расстояние от Санкт-Петербурга до Москвы равно (650±1) км. Найдите абсолютную и относительную погрешность.

ДПР Т1.2.

Обучающая часть работы.

Для того, чтобы привести число к стандартному виду, надо перенести в нём запятую так, чтобы она была сразу после первой значащей цифры, и полученное число умножить на 10k, где k подбирается так, чтобы произведение было равно данному числу.

В примерах в правых частях равенств записаны числа в стандартном виде. Напоминаем, что значащей цифрой числа называют его первую (слева направо) отличную от нуля цифру, а также все последующие за ней цифры.

Из определения стандартного вида числа следует, что в стандартном виде в целой части числа (до запятой) может содержаться только одна цифра. Все остальные цифры должны стоять после (справа от) запятой.

При решении задач числа округляют с точностью до первой, второй, третьей и т.д. значащей цифры. Запишем в стандартном виде и округлим радиус земного шара (6 370 000 м) до первой и второй значащей цифры:

6,37 · 106 м ≈ 6 · 106 м

6,37 · 106 м ≈ 6,4 · 106 м

Самостоятельная работа.

№1. Запишите число в стандартном виде:

  1. 73513
  2. 60396
  3. 0,5638
  4. 6348000
  5. 0,24+1
  6. 501,6
  7. 357*106

№2. Число молекул газа в 1 см3 при 0°С и давлении 760 мм.рс.ст равно 27 000 000 000 000 000 000. Записать это число в стандартном виде.

№3. 1 парсек (единица длины в астрономии) равен 30 800 000 000 000 км. Записать это число в стандартном виде.

№4. Сравните числа:

8,0382 · 106 и 1,099 · 1025;

1,76 · 103 и 2,5 · 10-4;

−1,3975 · 103 и −3,28 · 104;

−1,0015 · 10-8 и −1,001498 · 10-8.

ДПР Т1.3

Обучающая часть работы.

Комплексным числом в алгебраической форме называется выражение вида , где и действительные числа, а так называемая мнимая единица .

Действия и свойства действий над комплексными числами в алгебраической форме Определение1. Сложение комплексных чисел. Суммой двух комплексных чисел и называется комплексное число, определяемое равенством .

Определение 2. Вычитание комплексных чисел. Разностью двух комплексных чисел  и  называется такое комплексное число, которое, будучи сложенным с Z1, дает число Z2: .

Определение 3. Умножение комплексных чисел. Произведением комплексных чисел в алгебраической форме и  называется комплексное число, определяемое равенством  Эта формула формально получается путем перемножения двучленов  и  : .

Определение 4. Деление комплексных чисел. Деление комплексных чисел определяется как действие, обратное умножению. Практически деление комплексных чисел выполняется следующим образом: чтобы разделить  на , умножим числитель и знаменатель на число, сопряженное знаменателю (т.е. на ). Тогда делителем будет действительное число; разделив на него действительную и мнимую части делимого, получим частное:

Пример 1. Найти сумму и разность комплексных чисел  и

Пример 2. Умножение; .

Пример 3. Деление:

Самостоятельная работа.

№1. Найдите сумму комплексных чисел: Z1=-5+7i   и Z2=3+2i.

№2. Найдите разность комплексных чисел: Z1=-2+4i и Z2=5+9i.

№3. Найдите произведение комплексных чисел: Z1=10+11i и Z2=2+3i.

№4. Найдите частное комплексных чисел: Z1=-1+3i и Z2=4+5i.



Предварительный просмотр:

МИНИСТЕРСТВО ОБРАЗОВАНИЯ И НАУКИ РФ.
Федеральное агентство по образованию
Государственное образовательное учреждение профессионального образования
КАЗАНСКИЙСТРОИТЕЛЬНЫЙ КОЛЛДЕЖ.


ДПР-ДОМАШНИЕ ПРАКТИЧЕСКИЕ РАБОТЫ

I КУРС


Тема
«Корни, степени, логарифмы»

Составитель преподаватель
математики Садыкова Р.Н.

КАЗАНЬ

ДПР Т2.1

Вычисление и сравнение корней.

Обучающая часть.

Арифметическим корнем натуральной степени n ≥ 2 из неотрицательного числа a называется неотрицательное число, n-ая степень которого равна a, т.е.

, если  (а ≥ 0; b ≥ 0).

Свойства арифметического корня n-ой степени

a ≥ 0; b ≥ 0; n  N; m  N; k  N; n ≥ 2; m ≥ 2; k ≥ 2

1.;

2.;

3.;

4.;

5..

 Примеры.

1. Вычислить выражение:  

Решение: сначала упростим каждый из имеющихся корней:

,    ,            ,

,    ,    

После этого заданное выражение примет вид:

.

Ответ:

2.Упростить выражение , где a > 0, b > 0.

Решение: используя свойства арифметического корня, получаем

Ответ: ab

3.Сравните числа:

Пояснение:

68>63, значит

Самостоятельная работа:

1.Вычислить выражение:

6.

2.Упростить выражения:

1. , где a > 0, b > 0

2.

3.

4.

5.

6.

7.

8.

9.

10.

11.

12.

3. Сравните числа:

1. и

2. и

3.  и

и

 и

 и

 и

 и 7

 и 3100

 и 2

Вопросы.

1. Какое число называется арифметическим квадратным корнем?

2. Как называется число, стоящее под знаком корня?

3. Как читается выражение http://xn--i1abbnckbmcl9fb.xn--p1ai/%D1%81%D1%82%D0%B0%D1%82%D1%8C%D0%B8/620288/Image12330.gif?

4. Как называется действие нахождения квадратного корня из числа?

5. Из любого числа можно извлечь арифметический квадратный корень?

ДПР Т2.2

Вычисление и сравнение степеней.

Обучающая часть.

Степенью числа a>0 с рациональным показателем http://ykl-shk.azureedge.net/goods/ymk/algebra/work1/theory/2/10.gif,  где m - целое число, а n - натуральное ( n>1), называется число http://ykl-shk.azureedge.net/goods/ymk/algebra/work1/theory/2/11.gif, т.е.

http://ykl-shk.azureedge.net/goods/ymk/algebra/work1/theory/2/12.gif

http://ykl-shk.azureedge.net/goods/ymk/algebra/work1/theory/2/13.gif

Свойства степени с рациональным показателем.

http://ykl-shk.azureedge.net/goods/ymk/algebra/work1/theory/2/14.gif

Рекомендации к теме

При упрощении выражений, содержащих корни и степени с дробным показателем, можно переходить только к корням или только к степеням. Вы можете сами выбрать наиболее удобный для Вас путь решения задачи, я Вам рекомендую чаще пользоваться преобразованием выражений с помощью степени с дробным показателем и ее свойств. Свойства степеней имеют более простую форму и уменьшают вероятность совершения ошибки при преобразовании.

При выполнении упражнений на вычисление, упрощение выражений, содержащих степени с рациональным показателем, используют определение и свойства степени.

Сравнение степеней с одинаковыми основаниями

  • Если основание степени больше единицы (a>1), показательная функция возрастает, большему значению аргумента соответствует большее значение функции, соответственно, знак неравенства между показателями степеней и между степенями одинаковый.
  • Если основание степени меньше единицы (0

С помощью схемы сравнение степеней с равными основаниями можно изобразить так:

https://img-fotki.yandex.ru/get/15564/167134308.1f/0_12a169_408dfcfb_orig.png

Примеры.

1. Вычислить: http://ykl-shk.azureedge.net/goods/ymk/algebra/work1/recomend/2/15.gif

Решение: http://ykl-shk.azureedge.net/goods/ymk/algebra/work1/recomend/2/16.gif

Ответ: 5.

2. Упростить выражения: http://ykl-shk.azureedge.net/goods/ymk/algebra/work1/recomend/2/17.gif.

http://ykl-shk.azureedge.net/goods/ymk/algebra/work1/recomend/2/18.gif

Ответ: ab.

3. Упростить выражения:http://ykl-shk.azureedge.net/goods/ymk/algebra/work1/recomend/2/19.gif

Решение:

http://ykl-shk.azureedge.net/goods/ymk/algebra/work1/recomend/2/20.gif

Ответ: 2a.

4. Сравнить значения выражений:

  \[1){\left( {\frac{2}{7}} \right)^{1,5}}u{\left( {\frac{2}{7}} \right)^{1,9}}.\]

Решение:

Сравниваем показатели степеней: 1,5<1,9.

Основание a=2/7 меньше единицы, функция убывает, знак неравенства между степенями меняется на противоположный:

  \[{\left( {\frac{2}{7}} \right)^{1,5}} > {\left( {\frac{2}{7}} \right)^{1,9}}.\]

  \[2){(5,2)^{\sqrt 2 }}u{(5,2)^{\sqrt 3 }}.\]

Решение:

Сравниваем показатели степеней:

  \[\sqrt 2 < \sqrt 3 .\]

Основание a=5,2 больше единицы, функция возрастает, знак неравенства между степенями не меняется:

  \[{(5,2)^{\sqrt 2 }} < {(5,2)^{\sqrt 3 }}.\]

Самостоятельная работа.

1. Вычислить: 

1.

2. Упростить выражения: 

1.

2.

3.

4.

5.

6.

3. Сравнить значения выражений:

  1. 912 и 913
  2. 264 и 58
  3.  и
  4. 1020 и 2010
  5. 95 и
  6. и
  7. 273 и 36
  8.  и 28
  9. 452 – 312 и 442 – 302
  10. 2963 – 2143 и (296 – 214)3
  11. 20483 и 233
  12. 2816 и 7912
  13.  и

Вопросы:

1. Сформулируйте правило умножения степеней с одинаковым основанием.

2. Сформулируйте правило деления степеней с одинаковым основанием.

3. Сформулируйте правило возведения в степень степени.

4. Чему равна степень с нулевым показателем.

5. Каким числом положительным или отрицательным будет степень:

а) отрицательного числа с четным показателем;

б) положительного числа с четным показателем?

6. Каким числом положительным или отрицательным будет степень:

а) положительного числа с нечетным показателем;

б) отрицательного числа с нечетным показателем.

ДПР 2.3

Вычисление и сравнение логарифмов.

Обучающая часть.

Логарифмом положительного числа b по основанию http://ykl-shk.azureedge.net/goods/ymk/algebra/work5/theory/10/1.gif называется показатель степени с, в которую надо возвести число а, чтобы получить число b.

http://ykl-shk.azureedge.net/goods/ymk/algebra/work5/theory/10/2.gif

Основное логарифмическое тождество:

http://ykl-shk.azureedge.net/goods/ymk/algebra/work5/theory/10/3.gif

Свойства логарифмов:

http://ykl-shk.azureedge.net/goods/ymk/algebra/work5/theory/10/4.gif

7) Формула перехода к новому основанию:

http://ykl-shk.azureedge.net/goods/ymk/algebra/work5/theory/10/5.gif

Десятичный логарифм:

lga = log10a

Натуральный логарифм:

lna = logea, e ≈ 2,718…

При сравнении логарифмов используют свойства логарифмической функции

  \[y = {\log _a}x\]

При сравнении логарифмов с одинаковыми основаниями:

— если основание  больше единицы (a>1), функция возрастает, значит, большему значению аргумента соответствует большее значение функции (то есть знак неравенства не изменяется);

— если основание меньше единицы (0

С помощью схемы сравнение логарифмов можно изобразить так:

kak-sravnivat-logarifmy

Примеры.

1. Вычислить:

(3log72 – log724) : (log73 – log79).

Решение: Используя свойства логарифмов, получим

(3log72 – log724) : (log73 + log79)=(log723 – log724) : log727 = log73–1: log733 = – log73 : 3log73 =-(1/3).

Ответ: -1/3.

2.Вычислить: 4log25+2log0.253.

Решение: используя свойства степени, получим

4log25+2log0.253=4log25x42log0.253

 1) (22)log25=(2log25)2 =52=25

2)

http://ykl-shk.azureedge.net/goods/ymk/algebra/work5/recomend/10/6.gif

3) 25×1/9 = 25/9.

Ответ: 25/9.

3.Упростить:

1.

http://www.grandars.ru/images/1/review/id/1684/153c4f9a93.jpg

2.

 http://www.grandars.ru/images/1/review/id/1684/9ead9e8cf8.jpg

3.

http://www.grandars.ru/images/1/review/id/1684/7658468f48.jpg

4.

http://www.grandars.ru/images/1/review/id/1684/2247423abc.jpg

4.Сравнить:

  \[{\log _{81}}5\]   и    \[{\log _{\sqrt {27} }}6\]

Оба логарифма можно привести к основанию 3:

  \[{\log _{81}}5 = {\log _{{3^4}}}5 = \frac{1}{4}{\log _3}5 = \]

  \[ = {\log _3}{5^{\frac{1}{4}}} = {\log _3}\sqrt[4]{5},\]

  \[{\log _{\sqrt {27} }}6 = {\log _{{3^{\frac{3}{2}}}}}6 = \frac{1}{{\frac{3}{2}}}{\log _3}6 = \frac{2}{3}{\log _3}6 = \]

  \[ = {\log _3}{6^{\frac{2}{3}}} = {\log _3}\sqrt[3]{{{6^2}}} = {\log _3}\sqrt[3]{{36}}.\]

Так как

  \[\sqrt[4]{5} < \sqrt[3]{{36}}\]

и основание 3>1, функция возрастает и знак неравенства не изменяется:

  \[{\log _3}\sqrt[4]{5} < {\log _3}\sqrt[3]{{36}}\]

Следовательно,

  \[{\log _{81}}5 < {\log _{\sqrt {27} }}6.\]

Иногда бывает достаточно сравнить логарифмы с нулём.

Примеры.

Сравнить

  \[{\log _{1,4}}7,1u{\log _{0,9}}2,7\]

Сравним каждый из логарифмов с нулём:

  \[{\log _{1,4}}7,1 > 0,\]

  \[{\log _{0,9}}2,7 < 0.\]

Так как первый логарифм больше нуля, а второй — меньше нуля, то

  \[{\log _{1,4}}7,1 > {\log _{0,9}}2,7\]

3. Сравнить

  \[{\log _{\frac{1}{2}}}1,7\]   и    \[{\log _{2,6}}2,1\]

Сравниваем каждый из логарифмов с нулём:

  \[{\log _{\frac{1}{2}}}1,7 < 0,\]

  \[{\log _{5,6}}2,1 > 0\]

Первый логарифм меньше нуля, второй — больше нуля, следовательно, первый логарифм меньше второго:

  \[{\log _{\frac{1}{2}}}1,7 < {\log _{5,6}}2,1.\]

Сравнивать логарифмы можно, опираясь непосредственно на определение логарифма.

Например, сравним

  \[{\log _3}10\]   и     \[{\log _8}62.\]

  \[{3^2} = 9,{\log _3}10 > {\log _3}9, \Rightarrow {\log _3}10 > 2;\]

  \[{8^2} = 64,{\log _8}62 < {\log _8}64, \Rightarrow {\log _8}62 < 2.\]

Следовательно,

  \[{\log _3}10 > {\log _8}62.\]

Самостоятельная работа

1. Вычислить:

2. Упростить:

1.

2.

3.

4.

5.

6.

7.

8.

9.

10.

3.Сравнить:

 и

 и

 и lg2

5.lg1.05 и lg (1.05)-2

 и

 и lg20-lg3

 и

 и 4

 и

 и

ДПР Т2.4

Логарифмирование

Обучающая часть

Логарифмирование — действие, заключающееся в нахождении логарифма числа или выражения.

Логарифмирование является одним из двух действий, обратных возведению в степень. Если   \[{a^c} = b,\] то   \[a = \sqrt[c]{b},\]   \[c = {\log _a}b\]

Методом логарифмирования могут быть решены некоторые логарифмические уравнения.

Решение уравнения логарифмированием схематически можно описать приблизительно так.

  \[{(f(x))^{{{\log }_a}g(x)}} = b\]

ОДЗ:

  \[\left\{ \begin{array}{l} f(x) > 0;\\ g(x) > 0. \end{array} \right.\]

Логарифмируем обе части уравнения по основанию a:

  \[{\log _a}{(f(x))^{{{\log }_a}g(x)}} = {\log _a}b\]

(просто приписываем к обеим частям уравнения логарифм по основанию a. a — основание логарифма, стоящего в показателе степени).

Показатель степени выносим за знак логарифма:

  \[{\log _a}g(x) \cdot {\log _a}(f(x)) = {\log _a}b\]

Примеры решения уравнений методом логарифмирования.

  \[1){x^{{{\log }_3}x - 4}} = \frac{1}{{27}}\]

ОДЗ: x>0.

Логарифмируем обе части уравнения по основанию 3:

  \[{\log _3}{x^{{{\log }_3}x - 4}} = {\log _3}\frac{1}{{27}}\]

В левой части уравнения показатель степени выносим за знак логарифма. В правой части находим значение логарифма:

  \[({\log _3}x - 4) \cdot {\log _3}x = - 3\]

(Обратите внимание: показатель степени — разность. Сумму и разность при вынесении за знак логарифма обязательно нужно взять в скобки).

Полученное уравнение решаем с помощью замены переменной.

Пусть

  \[{\log _3}x = t,\]   тогда

  \[(t - 4) \cdot t = - 3\]

  \[{t^2} - 4t + 3 = 0\]

  \[{t_1} = 1;{t_2} = 3\]

Обратная замена:

  \[{\log _3}x = 1;{\log _3}x = 3\]

Эти простейшие логарифмические уравнения решаем по определению логарифма:

  \[{x_1} = {3^1};{x_2} = {3^3}\]

  \[{x_1} = 3;{x_2} = 27\]

Ответ: 1; 27.

  \[2){x^{{{\log }_2}x}} = 64x\]

ОДЗ: x>0.

Логарифмируем обе части уравнения по основанию 2:

  \[{\log _2}{x^{{{\log }_2}x}} = {\log _2}(64x)\]

(Обратите внимание: произведение в правой части уравнения записываем в скобках).

В левой части уравнения показатель степени выносим за знак логарифма. В правой части от логарифма произведения переходим к сумме логарифмов:

  \[{\log _2}x \cdot {\log _2}x = {\log _2}64 + {\log _2}x\]

  \[\log _2^2x = 6 + {\log _2}x\]

Пусть

  \[{\log _2}x = t,\]  тогда

  \[{t^2} - t - 6 = 0\]

  \[{t_1} = - 2;{t_2} = 3\]

Возвращаемся к исходной переменной:

  \[{\log _2}x = - 2;{\log _2}x = 3\]

  \[{x_1} = {2^{ - 2}};{x_2} = {2^3}\]

  \[{x_1} = \frac{1}{4};{x_2} = 8\]

Ответ: 1/4; 8.

  \[3){x^{3\lg x - \frac{1}{{\lg x}}}} = \sqrt[3]{{10}}\]

ОДЗ:

  \[\left\{ \begin{array}{l} x > 0;\\ \lg x \ne 0; \end{array} \right. \Rightarrow \left\{ \begin{array}{l} x > 0;\\ x \ne 1. \end{array} \right.\]

Прологарифмируем обе части уравнения по основанию 10:

  \[\lg {x^{3\lg x - \frac{1}{{\lg x}}}} = \lg \sqrt[3]{{10}}\]

В левой части показатель степени выносим за знак логарифма. Логарифм в правой части вычисляем:

  \[(3\lg x - \frac{1}{{\lg x}}) \cdot \lg x = \frac{1}{3}\]

Замена

  \[\lg x = t,(t \ne 0),\]

  \[(3t - \frac{1}{t}) \cdot t = \frac{1}{3}\]

  \[3{t^2} - 1 = \frac{1}{3}\]

  \[3{t^2} = \frac{4}{3}\]

  \[{t^2} = \frac{4}{9}\]

  \[t = \pm \frac{2}{3}\]

Обратная замена

  \[\lg x = \frac{2}{3};\lg x = - \frac{2}{3}\]

  \[{x_1} = {10^{\frac{2}{3}}};{x_2} = {10^{ - \frac{2}{3}}}\]

  \[{x_1} = \sqrt[3]{{100}};{x_2} = \frac{1}{{\sqrt[3]{{100}}}}\]

Ответ:

  \[\sqrt[3]{{100}};\frac{1}{{\sqrt[3]{{100}}}}.\]

  \[4){x^{{{\lg }^3}x - 5\lg x}} = 0,0001\]

ОЗД: x>0.

Прологарифмируем обе части уравнения по основанию 10:

  \[\lg {x^{{{\lg }^3}x - 5\lg x}} = \lg 0,0001\]

Показатель степени вынесем за знак логарифма

  \[({\lg ^3}x - 5\lg x) \cdot \lg x = - 4\]

Здесь сначала удобно раскрыть скобки

  \[{\lg ^4}x - 5{\lg ^2}x = - 4\]

Замена

  \[{\lg ^2}x = t,(t > 0),\]

  \[{t^2} - 5t + 4 = 0\]

  \[{t_1} = 1;{t_2} = 4\]

  \[{\lg ^2}x = 1;{\lg ^2}x = 4\]

  \[\lg x = 1;\lg x = - 1;\lg x = 2;\lg x = - 2\]

  \[{x_1} = 10;{x_2} = 0,1;{x_3} = 100;{x_4} = 0,01.\]

Самостоятельная работа



Предварительный просмотр:

МИНИСТЕРСТВО ОБРАЗОВАНИЯ И НАУКИ РФ.
Федеральное агентство по образованию
Государственное образовательное учреждение профессионального образования
КАЗАНСКИЙ СТРОИТЕЛЬНЫЙ КОЛЛДЕЖ.


ДПР-ДОМАШНЕ ПРАКТИЧЕСКИЕ РАБОТЫ

I КУРС


Тема
«Основы тригонометриии»

Составитель преподаватель
математики Садыкова Р.Н.

КАЗАНЬ

ДПР Т3.1

Радианный метод измерения углов вращения и связь с градусной мерой

Обучающая часть

Перевод градусов в радианы и обратно.

  1. 35°=
  2. sin 1,05°

Четверть углов

 четверть

-30°=  = = 4 четверть

Определение знака выражения:

Переводим все углы в градусную меру , затем смотрим в какой координатной четверти лежит полученное число .

=

sin135° 2 четверти(знак +)

cos150° 2 четверти(знак -)

Следу правилу «минус на плюс даёт минус» получаем :

<0

Самостоятельная работа

  1. Переведите радианную меру угла в градусную:

  ;   ;     ;   ;  3  ;  ;    .

  1. Определите в какой четверти лежит данный угол.

-120  ;  100;   145   ;  -200  ;    ;      .

  1.  
  1. Определите знак выражения  :

Практическая работа №2

Обучающая часть

Вычислите значение выражения

  1.  = cos(π+2×2π)

Упростить выражение ,используя  

Cos π =-1

  1. = -sin 600°

-

Упростить выражение ,используя  

-sin240°==

Вычислите значения синуса или косинуса

  1.    

=   >0

  1. 0

=  

  =  

Самостоятельная работа

Вычислите значение выражения

Вычислите значения синуса или косинуса


Практическая работа №3

Обучающая часть

Упростить выражение , используя

= 0

Упростить выражение , используя

 

Упростить выражение , используя

=  =  = 2

Самостоятельная работа



Предварительный просмотр:

МИНИСТЕРСТВО ОБРАЗОВАНИЯ И НАУКИ РФ.
Федеральное агентство по образованию
Государственное образовательное учреждение профессионального образования
КАЗАНСКИЙСТРОИТЕЛЬНЫЙ КОЛЛДЕЖ.


ДПР-ДОМАШНИЕ ПРАКТИЧЕСКИЕ РАБОТЫ

I КУРС


Тема
«Уравнения и неравенства»

Составитель преподаватель
математики Садыкова Р.Н.

КАЗАНЬ

ДП Т4.1

Обучающая часть работы.

Иррациональными уравнениями называются уравнения, в которых переменная содержится под знаком корня или знаком возведения в дробную степень. А вот как это выглядит:

​ Решение иррациональных уравнений обычно сводится к переходу от иррационального к рациональному уравнению путем возведения в степень n обеих частей уравнения.

При решении иррациональных уравнений необходимо учитывать следующее:

1. если показатель корня – четное число, то подкоренное выражение и значение корня не должны быть отрицательными;

2. если показатель корня – нечетное число, то подкоренное выражение может быть любым действительным числом;

3. при возведении обеих частей уравнения в четную степень  могут возникать посторонние корни, поэтому при использовании  данного метода необходимо делать проверку или находить область допустимых значений.

Пример:

1. Решить уравнение.

Решение:

ОДЗ.

3x−2≥0; 3x≥2 / : 3; x≥2\3

Возведём обе части уравнения в четвёртую степень.

Зx−2=16

3x=16+2

3x=18

x=6 ОДЗ

Ответ: x=6

Самостоятельная работа.

№1. Решите иррациональные уравнения:1.

2.

3.

4.

5.

6.

7.


ДПР Т4.2

Обучающая часть работа.

Найдём условие, при котором будут равны синусы двух углов. Пусть sin a = sin b. Тогда sin a – sin b = 0, и по известной формуле разности синусов имеем 

https://www.mathematics.ru/courses/algebra/content/javagifs/63261551616668-1.gif

Значит, либо https://www.mathematics.ru/courses/algebra/content/javagifs/63261551616668-2.gif то есть https://www.mathematics.ru/courses/algebra/content/javagifs/63261551616684-3.gif https://www.mathematics.ru/courses/algebra/content/javagifs/63261551616700-4.gif либо https://www.mathematics.ru/courses/algebra/content/javagifs/63261551616700-5.gif то есть https://www.mathematics.ru/courses/algebra/content/javagifs/63261551616731-6.gif https://www.mathematics.ru/courses/algebra/content/javagifs/63261551616731-7.gif Итак, sin a = sin b тогда и только тогда, когда либо a – b = 2πn, либо a + b = (2n + 1)π, https://www.mathematics.ru/courses/algebra/content/javagifs/63261551616746-8.gif

Рассмотрим решение простейшего уравнения sin x = a. Если |a| > 1, то решений нет, если |a| ≤ 1, то в силу периодичности синуса решений будет бесконечно много. По определению обратных тригонометрических функций, одно из решений − это arcsin a. Следовательно, наше уравнение можно переписать в виде sin x = sin (arcsin a). Тогда либо x – arcsin a = 2πnhttps://www.mathematics.ru/courses/algebra/content/javagifs/63261551616762-9.gif либо x + arcsin a = 2(n + 1)π, https://www.mathematics.ru/courses/algebra/content/javagifs/63261551616825-10.gif Оба эти равенства могут быть объединены в одно: 

https://www.mathematics.ru/courses/algebra/content/javagifs/63261551616840-11.gif

Это равенство называется формулой общего решения уравнения sin x = a, |a| ≤ 1.

Аналогично можно показать, что формула общего решения уравнения cos x = a при |a| ≤ 1 имеет вид 

https://www.mathematics.ru/courses/algebra/content/javagifs/63261551616887-12.gif

Формула общего решения уравнения tg x = a при любом действительном a имеет вид 

x = arctg a + πnhttps://www.mathematics.ru/courses/algebra/content/javagifs/63261551616950-13.gif

Формула общего решения уравнения ctg x = a при любом действительном a имеет вид 

x = arcctg a + πnhttps://www.mathematics.ru/courses/algebra/content/javagifs/63261551616965-14.gif

пример

Самостоятельная работа.

№1. Решите тригонометрические уравнения: 1.

2.

3.

4.

5.

6.

7.


ДПР Т 4.3

Обучающая часть работы.

Система уравнений — это условие, состоящее в одновременном выполнении нескольких уравнений относительно нескольких (или одной) переменных.

Если поставлена задача — найти такие пары значений (x;y), которые одновременно удовлетворяют уравнению p(x;y)=0 и уравнению q(x;y)=0, то говорят, что данные уравнения

образуют систему уравнений

{p(x;y)=0,q(x;y)=0.

Пару значений (x;y), которая одновременно является решением и первого и второго уравнений системы, называют решением системы уравнений.

Решить систему уравнений — значит найти все её решения или установить, что решений нет.

Может быть система и из трёх уравнений с тремя переменными:

Две системы уравнений называют равносильными, если они имеют одни и те же решения или если обе системы не имеют решений.

Для решения систем уравнений применяют методы:1. подстановки,2. алгебраического сложения,3. введения новых переменных,4. графический.

Пример:

Реши систему уравнений :

:

:

В ходе решения подставили вместо y выражение 3x−1, полученное из первого уравнения.

Введём во втором уравнении новую переменную

t=7x+1;

7x−1=7−(x+1) =t−1=1\t

Решая второе уравнение с переменной t, получим:

 t=7\t+6,

t 2 −6t−7=0, t≠0

t 1=−1,

t 2=7

Возвращаясь к введённому обозначению t, решаем полученные уравнения и находим x:

      7x+1=t

↙              ↘

7x+1=−1      7x+1=7=71

x∈∅               x+1=1

                       x=0

Найдём y, подставляя вместо x=0.

Получим, что y=−1.

Решение системы — пара чисел (0;−1).

В ходе решения были использованы два метода: подстановки и введение новой переменной

Самостоятельная работа

  1. №1. Решите системы уравнений:1. 3.

4.  5. 6.7.

ДПР Т4.4

Обучающая часть.

  1. Неравенство, в котором неизвестная переменная находится под знаком тригонометрической функции, называется тригонометрическим неравенством.
  2. К простейшим тригонометрически неравенствам относятся следующие 16 неравенств: 
    sinx >a, sinx ≥a, sinxcosx>a, cosx≥a, cosxtanx>a, tanx≥a, tanxcotx>a, cotx≥a, cotxЗдесь x является неизвестной переменной, a может быть любым действительным числом.

   Неравенства вида   sinx>asinx≥asinxsinx≤a

решения простейших неравенств с функцией синус

Рис.1

Рис.2

   Неравенство sinx>a

  1. При |a|≥1 неравенство sinx>a не имеет решений: 
    x
    ∈∅
  2. При a<−1 решением неравенства sinx>a является любое действительное число: 
    x
    R
  3. При −1≤a<1 решение неравенства sinx>a выражается в виде 
    arcsina+2πnZ  (рис.1). 

   Неравенство sinx≥a

  1. При a>1 неравенство sinx≥a не имеет решений: 
    x
    ∈∅
  2. При a≤−1 решением неравенства sinx≥a является любое действительное число: 
    x
    R
  3. Случай a=1 
    x=π/2+2πn,n
    Z
  4. При −1arcsina+2πn≤x≤π−arcsina+2πn,nZ  (рис.1). 

   Неравенство sinx

  1. При a>1 решением неравенства sinxxR
  2. При a≤−1 у неравенства sinxx∈∅
  3. При −1−π−arcsina+2πnZ  (рис.2). 

   Неравенство sinx≤a

  1. При a≥1 решением неравенства sinx≤a является любое действительное число: 
    x
    R
  2. При a<−1 неравенство sinx≤a решений не имеет: 
    x
    ∈∅
  3. Случай a=−1 
    x=−π/2+2πn,n
    Z
  4. При −1−π−arcsina+2πn≤x≤arcsina+2πn,nZ  (рис.2). 

   Неравенства вида   cosx>acosx≥acosxcosx≤a

решения простейших неравенств с функцией косинус

Рис.3

Рис.4

   Неравенство cosx>a

  1. При a≥1 неравенство cosx>a не имеет решений: 
    x
    ∈∅
  2. При a<−1 решением неравенства cosx>a является любое действительное число: 
    x
    R
  3. При −1≤a<1 решение неравенства cosx>a имеет вид 
    −arccosa+2πnZ  (рис.3). 

   Неравенство cosx≥a

  1. При a>1 неравенство cosx≥a не имеет решений: 
    x
    ∈∅
  2. При a≤−1 решением неравенства cosx≥a является любое действительное число: 
    x
    R
  3. Случай a=1 
    x=2πn,n
  4. При −1−arccosa+2πn≤x≤arccosa+2πn,nZ  (рис.3). 

   Неравенство cosx

  1. При a>1 неравенство cosxxR
  2. При a≤−1 неравенство cosxx∈∅
  3. При −1arccosa+2πnZ  (рис.4). 

   Неравенство cosx≤a

  1. При a≥1 решением неравенства cosx≤a является любое действительное число: 
    x
    R
  2. При a<−1 неравенство cosx≤a не имеет решений: 
    x
    ∈∅
  3. Случай a=−1 
    x=π+2πn,n
    Z
  4. При −1arccosa+2πn≤x≤2π−arccosa+2πn,nZ  (рис.4). 

   Неравенства вида   tanx>atanx≥atanxtanx≤a

решения простейших неравенств с функцией тангенс

Рис.5

Рис.6

   Неравенство tanx>a

  1. При любом действительном значении a решение строгого неравенства tanx>a имеет вид 
    arctana+πnZ  (рис.5). 

   Неравенство tanx≥a

  1. Для любого значения a решение неравенства tanx≥a выражается в виде 
    arctana+πn≤x<π/2+πn,n
    Z  (рис.5). 

   Неравенство tanx

  1. Для любого значения a решение неравенства tanx−π/2+πnZ  (рис.6). 

   Неравенство tanx≤a

  1. При любом a неравенство tanx≤a имеет следующее решение: 
    −π/2+πnZ  (рис.6). 

   Неравенства вида  cotx>acotx≥acotxcotx≤a

решения простейших неравенств с функцией котангенс

Рис.7

Рис.8

   Неравенство cotx>a

  1. При любом a решение неравенства cotx>a имеет вид 
    πnZ  (рис.7). 

   Неравенство cotx≥a

  1. Нестрогое неравенство cotx≥a имеет аналогичное решение 
    πnZ  (рис.7). 

   Неравенство cotx

  1. Для любого значения a решение неравенства cotxarccot a+πnZ  (рис.8). 

   Неравенство cotx≤a

  1. При любом a решение нестрогого неравенства cotx≤a находится в полуоткрытом интервале 
    arccot a+πn≤x<π+πn,n
    Z  (рис.8). 

П р и м е р  1 .  Решить неравенство:   sin x > 0.

Р е ш е н и е .  В пределах одного оборота единичного радиуса это неравенство

                         справедливо при 0 < http://www.bymath.net/studyguide/pi.gif. Теперь необходимо добавить период

                         синуса  2http://www.bymath.net/studyguide/pi.gif n :

                                        http://www.bymath.net/studyguide/tri/sec/tri18g.gif

П р и м е р  2 .  Решить неравенство:   sin x > 0.5 .

Р е ш е н и е .

                            http://www.bymath.net/studyguide/tri/sec/tri18a.gif

http://www.bymath.net/studyguide/tri/sec/tri18h.gif

http://www.bymath.net/studyguide/tri/sec/tri18b.gif

П р и м е р  4 .  Решить систему неравенств:

http://www.bymath.net/studyguide/tri/sec/tri18d.gif

 Второе неравенство tan x < 1  имеет решение:

http://www.bymath.net/studyguide/tri/sec/tri18f.gif

Самостоятельная работа.

  1. Sinx >http://xn--i1abbnckbmcl9fb.xn--p1ai/%D1%81%D1%82%D0%B0%D1%82%D1%8C%D0%B8/585357/img21.gif

  1. соsx >http://xn--i1abbnckbmcl9fb.xn--p1ai/%D1%81%D1%82%D0%B0%D1%82%D1%8C%D0%B8/585357/img23.gif

               √2

  1. sin 3x ≤——

                 2

                     1

  1. cos 2x  ≥  – —

                     2

Практическая работа № 5

Обучающая часть работы.

Решение тригонометрических неравенств

Решение тригонометрических неравенств зачастую сводится к решению простейших тригонометрических неравенств вида:

sinx >a, sinx ≥a, sinxcosx>a, cosx≥a, cosxtanx>a, tanx≥a, tanxcotx>a, cotx≥a, cotx

Решаются простейшие тригонометрические неравенства графически или с помощью единичной тригонометрической окружности.

По определению, синусом угла \alpha есть ординатой точки {{P}_{\alpha }}\left( x,y \right) единичного круга (рис. 1), а косинусом – абсцисса этой точки. Этот факт используется при решении простейших тригонометрических неравенств с косинусом и синусом с помощью единичного круга.

http://ru.solverbook.com/my_images/677.png

Рис. 1

Примеры

1. Найти значение выражения:hello_html_505fec51.gif

Решение:

hello_html_190a9b80.gif

2. Решить уравнение: hello_html_500f5de6.gif

Решение:

hello_html_m1c93a554.gif

hello_html_5789b386.gif

hello_html_m1cf2d47c.gif

hello_html_m12a4afb2.gif

hello_html_m6e259c2f.gif

hello_html_5fc39c56.gif

Ответ: hello_html_5fc39c56.gif

3. Решить уравнение: hello_html_m621a98bf.gif

Решение:

hello_html_2b8033d8.gif

hello_html_4f8dc397.gif

hello_html_39746ee1.gif

hello_html_m5ba4e8ba.gif

hello_html_m6268b062.gif

hello_html_4f6e66a9.gif

hello_html_m790e54f7.gif

Ответ: hello_html_m790e54f7.gif

4. Решить уравнение: hello_html_m62c982ea.gif hello_html_11852162.gif

Решение:

hello_html_71f0c77f.gif

hello_html_m78f408ed.gif

hello_html_524203.gif

hello_html_m1aac29b9.gif

hello_html_4a61655f.gif

hello_html_211b9bfd.gif

hello_html_4c947d8f.gif                               Ответ: hello_html_40b7ae8.gif

Самостоятельная работа.

  1. cosx+2cos2x=1                                                  
  2. 2cos2+4cosx=3sin2x
  3. cos2x +4sin2x=2sin2x                                      
  4. 2cos2+4cosx=3sin2x

Практическая работа №6

Обучающая часть работа.

ЛОГАРИФМИЧЕСКИЕ НЕРАВЕНСТВА

ОПРЕДЕЛЕНИЕ

Неравенства, которые содержат переменную под знаком логарифма или в его основании, называются логарифмическими.

Решение логарифмических неравенств основывается на свойстве монотонности логарифмической функции: функция y=logax монотонно возрастает, если a>1 , и монотонно убывает, если 0

Logaf(x)>loga9(x)

при потенцировании, для значений a>1 знак неравенства сохраняется; а для значений 0

В случае если переменная содержится и в основании, и в подлогарифмическом выражении, например  , решение разбивается два случая, когда  и, когда  , то есть

Пример

Самостоятельная работа. 

Практическая работа №7

Обучающая часть работа.

Показательные неравенства

Показательными называются неравенства, в которых неизвестная переменная содержится только в показателях каких-либо степеней.

Для решения показательных неравенств требуется знание следующей теоремы:

Теорема 2. Если a > 1, то неравенство af(x) > ag(x) равносильно неравенству того же смысла: f(x) > g(x). Если 0 < a < 1, то показательное неравенство af(x) > ag(x) равносильно неравенству противоположного смысла: f(x) < g(x).

Пример 1. Решите неравенство: 16^x-2\cdot 12^x\leqslant 3^{2x+1}.

Решение: представим исходное неравенство в виде: 4^{2x}-2\cdot 4^x\cdot 3^x-3\cdot 3^{2x}\leqslant 0.

Разделим обе части этого неравенства на 32x, при этом (в силу положительности функции = 32x) знак неравенства не изменится: \left(\frac{4}{3}\right)^{2x}-2\cdot\left(\frac{4}{3}\right)^x-3\leqslant 0.

Воспользуемся подстановкой: t=\left(\frac{4}{3}\right)^x.

Тогда неравенство примет вид: ~t^2-2t-3\leqslant 0.

Решение неравенства t^2-2t-3<0

Решение неравенства на числовой прямой

Итак, решением неравенства является промежуток:

-1\leqslant t\leqslant 3,

переходя к обратной подстановке, получаем:

-1\leqslant \left(\frac{4}{3}\right)^x \leqslant 3.

Левое неравенства в силу положительности показательной функции выполняется автоматически. Воспользовавшись известным свойством логарифма, переходим к эквивалентному неравенству:

\left(\frac{4}{3}\right)^x \leqslant \left(\frac{4}{3}\right)^{\log_{\frac{4}{3}}3}.

Поскольку в основании степени стоит число, большее единицы, эквивалентным (по теореме 2) будет переход к следующему неравенству:

x\leqslant \log_{\frac{4}{3}}3.

Итак, окончательно получаем ответ: x\in(-\mathcal{1};\log_{\frac{4}{3}}3].

Пример 2. Решите неравенство: \frac{7^x-30}{7^{x-1}+1}\leqslant -14.

Решение: используя свойства умножения и деления степеней, перепишем неравенство в виде:

\frac{7^x-30}{\frac{1}{7}\cdot 7^x+1}\leqslant -14.

Введем новую переменную: ~t=7^x.

С учетом этой подстановки неравенство принимает вид: \frac{t-30}{\frac{1}{7}\cdot t+1}+14\leqslant 0\Leftrightarrow

Умножим числитель и знаменатель дроби на 7, получаем следующее равносильное неравенство:

\frac{7t-210}{t+7}+14\leqslant 0\Leftrightarrow\frac{21t-112}{t+7}\leqslant 0\Leftrightarrow \frac{3t-16}{t+7}\leqslant 0.

Числовой промежуток, соответствующей решению неравенства (3t-16)/(t+7) <= 0

Итак, неравенству удовлетворяют следующие значения переменной t:

-7\leqslant t\leqslant \frac{16}{3}.

Тогда, переходя к обратной подстановке, получаем:

7\leqslant 7^x\leqslant \frac{16}{3}.

7^x\leqslant 7^{\log_7 \frac{16}{3}}.

Поскольку основание степени здесь больше единицы, равносильным (по теореме 2) будет переход к неравенству: x\leqslant \log_7 \frac{16}{3}.

Окончательно получаем ответ:

x\in\left(-\mathcal{1};\log_7 \frac{16}{3}\right].

Пример 3. Решите неравенство:

\[ 2^{2x^2-6x+3}+6^{x^2-3x+1}-3^{2x^2-6x+3}\geqslant 0. \]

Решение:

\[ 2\cdot 2^{2x^2-6x+2} + 2^{x^2-3x+1}\cdot 3^{x^2-3x+1} - 3\cdot 3^{2x^2-6x+2}\geqslant 0\Leftrightarrow \]

Делим обе части неравенства на выражение:

\[ 3^{2x^2-6x+2}. \]

Оно всегда больше нуля (из-за положительности показательной функции), поэтому знак неравенства изменять не нужно. Получаем: \[ 2\cdot\left(\frac{2}{3}\right)^{2x^2-6x+2}+\left(\frac{2}{3}\right)^{x^2-3x+1}-3\geqslant 0. \]

Воспользуемся заменой переменной:

\[ t = \left(\frac{2}{3}\right)^{x^2-3x+1}. \]

Исходное уравнение тогда принимает вид: \[ 2t^2+t-3 \geqslant 0. \]

Решение неравенства 2t^2+t-3>=0

Числовой промежуток, являющийся решением неравенства 2t^2+t-3>=0

Итак, неравенству удовлетворяют значения t, находящиеся в промежутке:

\[ t\in\left(-\mathcal{1};-\frac{3}{2}\right]\cup[1;+\mathcal{1}). \]

Переходя к обратной подстановке получаем, что исходное неравенство распадается на два случая:

\[ \left[\begin{matrix}\left(\frac{2}{3}\right)^{x^2-3x+1}\leqslant -\frac{3}{2}, \\ \left(\frac{2}{3}\right)^{x^2-3x+1}\geqslant 1\end{matrix}\right. \]

Первое неравенство решений не имеет в силу положительности показательной функции. Решаем второе:

\[ \left(\frac{2}{3}\right)^{x^2-3x+1}\geqslant 1\Leftrightarrow \left(\frac{2}{3}\right)^{x^2-3x+1}\geqslant \left(\frac{2}{3}\right)^{0} \]

Поскольку основание степени в данном случае оказалось меньше единицы, но больше нуля, равносильным (по теореме 2) будет переход к следующему неравенству:

\[ x^2-3x+1\leqslant 0. \]

Графическое решение неравенства x^2-3x+2<=0

Числовой промежуток, соответствующий решению неравенства x^2-3x+1\leqslant 0

Итак, окончательный ответ:

\[ x\in\left[\frac{3-\sqrt{5}}{2};\frac{3+\sqrt{5}}{2}\right]. \]

Самостоятельная работа.


Предварительный просмотр:


Предварительный просмотр:

МИНИСТЕРСТВО ОБРАЗОВАНИЯ И НАУКИ РФ.
Федеральное агентство по образованию
Государственное образовательное учреждение профессионального образования
КАЗАНСКИЙ СТРОИТЕЛЬНЫЙ КОЛЛДЕЖ.


ДПР-ОМАШНЯЯ ПРАКТИЧЕСКАЯ РАБОТА

I КУРС


Тема
«ПРЯМЫЕ И ПЛОСКОСТИ В ПРОСТРАНСТВЕ»

Составитель преподаватель
математики Садыкова Р.Н.

КАЗАНЬ

ДПР Т6.1

Взаимное расположение прямых

Угол между прямыми


Предварительный просмотр:


Предварительный просмотр:

Федеральное агентство по образованию
Государственное образовательное учреждение профессионального образования
КАЗАНСКИЙСТРОИТЕЛЬНЫЙ КОЛЛДЕЖ.


ПРАКТИЧЕСКИЕ РАБОТЫ.

I КУРС


Тема
«Начало математического анализа»

Составитель преподаватель
математики Садыкова Р.Н.

КАЗАНЬ-2017

Практическая работа № 1        

Вычисление производной.

Цель: Сформировать умения находить производные функций, заданных в явном, логарифмическом и параметрическом виде, находить производные сложных функций, геометрический смысл производной, применять правило Лопиталя для нахождения пределов.

Теоретические сведения к практической работе

Производной функции  называется конечный предел отношения приращения функции  к приращению независимой переменной  при стремлении последнего к нулю:

                                               (1)

   Обозначения производной в точке х0:

 и другие.

Если функция в точке х0 (или на промежутке Х) имеет конечную производную, то функция называется дифференцируемой в этой точке (или на промежутке Х).

Процесс отыскания производной называется дифференцированием.


Геометрический смысл производной.

Если кривая задана уравнением , то — угловой коэффициент касательной к графику функции в этой точке ().

Уравнение касательной к кривой  в точке х0 (прямая М0Т) имеет вид:

                                                                      (2)

а уравнение нормали (М0N):

                                                                    (3)

Правила дифференцирования

№ пп

U = u(x),    V=V(x) —
дифференцируемые функции

№ пп

U = u(x),    V=V(x) —
дифференцируемые функции

I

VI

Производная сложной функции

II

VII

Функция задана параметричес-кими уравнениями

III

IV

VIII

Если  и  —
взаимно обратные функции,
то

V

Формулы дифференцирования основных элементарных функций

№ пп

 с=const, х — независимая переменная,
u = u(x) — дифференцируемая функция

1

С’= 0

9

2

x’= 1

10

3

11

4

12

5

13

6

14

7

15

8

Производной n-го порядка называется производная от производной (n–1)-го порядка. Производные высших порядков вычисляются последовательным дифференцированием данной функции.

Производная второго порядка  или

Производная третьего порядка  или  и т. д.

Пример 1. Найти производные функций:

а)  б)  в)  г)

Решение.

а) Используя правила I, III и формулу (3), получим:

б) Используя правила дифференцирования произведения функций II, разности I, формулы (5), (7), (8) и учитывая, что независимая переменная есть t, т. е. t=1, получим:

в) Сложная степенная функция, независимая переменная есть v,
т. е. v=1; используя формулу (3), получим:

г) Используя правила дифференцирования частного IV, суммы I, III
и формулы (3), (14), учитывая, что t=1, получим:

Пример 2. Составить уравнение касательной и нормали к кривой в точке с абсциссой х0=2.

Используем уравнения касательной (2) и нормали (3):

1)

2)

   

Подставим  в уравнения и получим:

или  — уравнение касательной.

 или — уравнение нормали.

Пример 3. Найти производную , если функция задана парамет-рически:

Используем правило VII


Пример 4. Найти дифференциалы функций:

а)  б)  в)

Для дифференциала функции  справедлива формула  т. е. дифференциал функции равен произведению производной от функции на дифференциал независимой переменной.

Решение.

а)

б)

в)

Пример 5. Найти производную второго порядка функции

Решение.  поэтому найдём производную первого порядка,
а затем второго.

Пример 6. Найти производную функции  логарифмическим дифференцированием

Задания для самостоятельной работы

Задание 1. Найти производные 1-го порядка данных функций

1)         

2) 3)         

4)

5)

6)         



Предварительный просмотр:

Федеральное агентство по образованию
Государственное образовательное учреждение профессионального образования
КАЗАНСКИЙСТРОИТЕЛЬНЫЙ КОЛЛДЕЖ.


ПРАКТИЧЕСКИЕ РАБОТЫ.

I КУРС


Тема
«Интегралы и его применение»

Составитель преподаватель
математики Садыкова Р.Н.

КАЗАНЬ-2017

Практическая работа №

Вычисление определенных интегралов

Цель: Научиться вычислять определенные интегралы, используя различные методы интегрирования.

Теоретические сведения к практической работе

Таблица основных интегралов

1.                            2.

3.                  

4.                                        5.

6.                                         7.

8.                              9.

10.                               11.

12.          13.

14.                    15.

16.                    17.

18.

Определенный интеграл, его вычисление и свойства

Определенный интеграл от функции, непрерывной на отрезке , вычисляется по формуле:

        (5)

где — первообразная для функции , т. е.

Формула (5) называется формулой Ньютона — Лейбница.

Свойства определенного интеграла:

               

6) Если  для всех , то

7) Если  для всех , то

При вычислении определенного интеграла для нахождения первообразной используют те же методы, что и для нахождения неопределенного интеграла, т. е. замену переменной, интегрирование по частям и т. д. Однако есть ряд особенностей. При замене переменной по формуле (1) необходимо в соответствии с заменой менять пределы интегрирования:

        (6)

где  — обратная к  функция.

Формула интегрирования по частям (3) приобретает вид:

        (7)

        Пример 4. Вычислить определенный интеграл

Решение.

 

Содержание практической работы

Вычислить определенный интеграл.

1)

2)

3)

4)

5)

6)


Практическая работа №6

Вычисление площадей плоских фигур.

Цель: Применять определенный интеграл для вычисления площадей, длин и объемов фигур.

Теоретические сведения к практической работе

Площади плоских фигур

1. Вычисление площадей плоских фигур в декартовой системе координат

Если плоская фигура (рис. 1) ограничена линиями  , где  для всех , и прямыми , , то ее площадь вычисляется по формуле:

        (8)

рис

рис2_ред1

Рис. 1

Рис. 2

Пример. Найти площадь фигуры, ограниченной линиями:

Решение. Построим схематический рисунок (рис. 2). Для построения параболы возьмем несколько точек:

x

0

1

–1

2

–2

3

–3

4

–4

y

–2

–1

–1

2

2

7

7

14

14

Для построения прямой достаточно двух точек, например  и .

Найдем координаты точек  и  пересечения параболы  и прямой .

Для этого решим систему уравнений

Тогда  Итак,

Площадь полученной фигуры найдем по формуле (8), в которой

 поскольку  для всех . Получим:

2. Вычисление площадей фигур, ограниченных линиями, заданными параметрически

Если функции  и  имеют непрерывные производные первого порядка для всех , то площадь плоской фигуры, ограниченной линией  прямыми  x = a, x = b, где  a = x(t0),

b = x(t1),  и осью OX,  вычисляется по формуле:

        (9)

Пример. Найти площадь фигуры, ограниченной линиями, заданными параметрически:

Решение. Для построения фигуры составим таблицу значений координат (x, y) точек кривой, соответствующих различным значениям параметра

t

0

x

2

0

–2

0

2

y

0

3

0

–3

0

рис

Рис. 3

Нанесем точки (x, y) на координатную плоскость XOY и соединим плавной линией. Когда параметр  изменяется от  до , соответствующая точка  описывает эллипс (известно, что  — параметрические формулы, задающие эллипс с полуосями a и b). Учитывая симметрию фигуры относительно координатных осей OX и OY, найдем её площадь S, умножив на 4 площадь криволинейной трапеции AOB. Согласно формуле (9) получим:

Длина дуги плоской кривой

1. Вычисление дуги плоской кривой в декартовых координатах

рис

Рис. 4

Если кривая задана уравнением , функция  имеет непрерывную первую производную при всех , то длина дуги  (рис. 4) этой кривой, заключенной между точками  и , вычисляется по формуле:

        (10)

2. Вычисление длины дуги кривой, заданной параметрически

Если кривая задана параметрически , и функции  имеют непрерывные производные 1-го порядка при всех , то длина дуги ,  соответствующей изменению параметра от  до , вычисляется по формуле:

        (11)

Пример. Найти длину дуги кривой

а)  б)

Решение.

а) Так как кривая задана в декартовой системе координат уравнением , то для вычисления длины дуги воспользуемся формулой (10). Найдем :                 и подставим в (10):

б)

Кривая задана параметрически, поэтому воспользуемся формулой (11). Найдем :

и подставим в (11):

Вычисление объемов тел вращения

Если тело образовано вращением вокруг оси OX криволинейной трапеции, ограниченной кривой , осью OX и прямыми ,  (рис. 5), то его объем вычисляется по формуле:

        (12)

рис

рис

Рис. 5

Рис. 6

        Пример. Найти объем тела, полученного вращением вокруг оси OX фигуры, ограниченной линиями:

Решение. Построим криволинейную трапецию, вращением которой получается тело вращения (рис. 6).

Чтобы получить объем тела вращения из объема  тела, полученного вращением фигуры ОАВС, вычтем объем  тела, полученного вращением фигуры ОАВ. Тогда искомый объем . По формуле (12) найдем  и :       (ед. объема);

 (ед. объема);

(ед. объема).

Содержание практической работы

Задание 1. Найти площадь фигуры, ограниченной линиями.

1)

2)

3)

4)

5)

6)

Задание 2. Найти площадь фигуры, ограниченной линиями, заданными параметрически.

1)

2)

3)

4)

5)

6)

Задание 3. Найти длину дуги кривой.

1)

2)

3)

4)

5)

6)

Задание 4. Найти объем тела, полученного вращением вокруг оси OX фигуры, ограниченной линиями.

1)

2)

3)

4)

5)

6)




Предварительный просмотр:

МИНИСТЕРСТВО ОБРАЗОВАНИЯ И НАУКИ РФ.
Федеральное агентство по образованию
Государственное образовательное учреждение профессионального образования
КАЗАНСКИЙСТРОИТЕЛЬНЫЙ КОЛЛДЕЖ.


ПРАКТИЧЕСКИЕ РАБОТЫ

I КУРС


Тема
«Координаты и векторы»

Составитель преподаватель
математики Садыкова Р.Н.

КАЗАНЬ-2017

Обучающая часть

Понятие вектора

Векторы занимают особое место среди объектов, рассматриваемых в высшей математике, поскольку каждый вектор имеет не только числовое значение - длину, но и физическое и геометрическое - направленность. Вектор, представленный направленным отрезком, идущим от точки A к точке B, обозначается так: https://function-x.ru/vectors/v01.gif.

https://function-x.ru/image/vect_main.jpg

Вектор - это вид представления точки, до которой требуется добраться из некоторой начальной точки. Например, трёхмерный вектор, как правило, записывается в виде (х, y, z). Говоря совсем просто, эти числа означают, как далеко требуется пройти в трёх различных направлениях, чтобы добраться до точки.

Пусть дан вектор. При этом x = 3 (правая рука указывает направо), y = 1(левая рука указывает вперёд), z = 5 (под точкой стоит лестница, ведущая вверх). По этим данным вы найдёте точку, проходя 3 метра в направлении, указываемом правой рукой, затем 1 метр в направлении, указываемом левой рукой, а далее Вас ждёт лестница и, поднимаясь на 5 метров, Вы, наконец, окажетесь в искомой точке.


Физическими примерами векторных величин могут служить смещение материальной точки, двигающейся в пространстве, скорость и ускорение этой точки, а также действующая на неё сила.


Геометрический вектор представлен в двумерном и трёхмерном пространстве в виде направленного отрезка, т.е. отрезка, у которого различают начало и конец.https://function-x.ru/image/vectors1.jpg

Если A - начало вектора, а B - его конец, то вектор обозначается символом https://function-x.ru/vectors/v01.gif или одной строчной буквой https://function-x.ru/vectors/v02.gif. На рисунке конец вектора указывается стрелкой (рис. 1)


Длиной (или модулем) геометрического вектора https://function-x.ru/vectors/v01.gif называется длина порождающего его отрезка https://function-x.ru/vectors/v03.gif


Два вектора называются равными, если они могут быть совмещены (при совпадении направлений) путём параллельного переноса, т.е. если они параллельны, направлены в одну и ту же сторону и имеют равные длины.


Линейные операции над геометрическими векторами

Умножение вектора на числоhttps://function-x.ru/image/vectors2.jpg

Произведением вектора https://function-x.ru/vectors/v02.gif на число https://function-x.ru/vectors/v04.gifназывается вектор, получающийся из вектора https://function-x.ru/vectors/v02.gif растяжением (при https://function-x.ru/vectors/v05.gif) или сжатием (при https://function-x.ru/vectors/v06.gif) в https://function-x.ru/vectors/v07.gif раз, причём направление вектора https://function-x.ru/vectors/v02.gif сохраняется, если https://function-x.ru/vectors/v08.gif, и меняется на противоположное, если https://function-x.ru/vectors/v09.gif. (Рис. 2)

Из определения следует, что векторы https://function-x.ru/vectors/v02.gifи https://function-x.ru/vectors/v10.gif = https://function-x.ru/vectors/v11.gif всегда расположены на одной или на параллельных прямых. Такие векторы называются коллинеарными. (Можно говорить также, что эти векторы параллельны, однако в векторной алгебре принято говорить "коллинеарны".) Справедливо и обратное утверждение: если векторы https://function-x.ru/vectors/v02.gif и https://function-x.ru/vectors/v10.gif коллинеарны, то они связаны отношением

https://function-x.ru/vectors/v12.gif.   (1)

Следовательно, равенство (1) выражает условие коллинеарности двух векторов.

https://function-x.ru/image/vectors3.jpg


Сложение и вычитание векторов

При сложении векторов нужно знать, что суммой векторов https://function-x.ru/vectors/v02.gif и https://function-x.ru/vectors/v10.gif называется вектор https://function-x.ru/vectors/v13.gif, начало которого совпадает с началом вектора https://function-x.ru/vectors/v02.gif, а конец - с концом вектора https://function-x.ru/vectors/v10.gif, при условии, что начало вектора https://function-x.ru/vectors/v10.gif приложено к концу вектора https://function-x.ru/vectors/v02.gif. (Рис. 3)

https://function-x.ru/image/vectors4.jpg

Это определение может быть распределено на любое конечное число векторов. Пусть в пространстве даны n свободных векторов https://function-x.ru/vectors/v14.gif. При сложении нескольких векторов за их сумму принимают замыкающий вектор, начало которого совпадает с началом первого вектора, а конец - с концом последнего вектора. То есть, если к концу вектора https://function-x.ru/vectors/v15.gif приложить начало вектора https://function-x.ru/vectors/v16.gif, а к концу вектора https://function-x.ru/vectors/v16.gif - начало вектора https://function-x.ru/vectors/v17.gif и т.д. и, наконец, к концу вектора https://function-x.ru/vectors/v18.gif - начало вектора https://function-x.ru/vectors/v19.gif, то суммой этих векторов служит замыкающий вектор https://function-x.ru/vectors/v20.gif, начало которого совпадает с началом первого вектора https://function-x.ru/vectors/v15.gif, а конец - с концом последнего вектора https://function-x.ru/vectors/v19.gif. (Рис. 4)

Слагаемые https://function-x.ru/vectors/v14.gif называются составляющими вектора https://function-x.ru/vectors/v21.gif, а сформулированное правило - правилом многоугольника. Этот многоугольник может и не быть плоским.

При умножении вектора https://function-x.ru/vectors/v02.gif на число -1 получается противоположный вектор https://function-x.ru/vectors/v22.gif. Векторы https://function-x.ru/vectors/v02.gif и https://function-x.ru/vectors/v22.gif имеют одинаковые длины и противоположные направления. Их сумма https://function-x.ru/vectors/v23.gif даёт нулевой вектор, длина которого равна нулю. Направление нулевого вектора не определено.

В векторной алгебре нет необходимости рассматривать отдельно операцию вычитания: вычесть из вектора https://function-x.ru/vectors/v02.gif вектор https://function-x.ru/vectors/v10.gif означает прибавить к вектору https://function-x.ru/vectors/v02.gifпротивоположный вектор https://function-x.ru/vectors/v24.gif, т.е. https://function-x.ru/vectors/v25.gif

Пример 1. Упростить выражение:

https://function-x.ru/vectors/v140.gif.

Решение:

https://function-x.ru/vectors/v141.gif,

то есть, векторы можно складывать и умножать на числа так же, как и многочлены (в частности, также задачи на упрощение выражений). Обычно необходимость упрощать линейно подобные выражения с векторами возникает перед вычислением произведений векторов.

Проекция вектора на ось

Проекция вектора на ось равна произведению длины проектируемого вектора на косинус угла между вектором и осью:

https://function-x.ru/vectors/v32.gif

Как известно, проекцией точки A на прямую (плоскость) служит основание https://function-x.ru/vectors/v26.gifперпендикуляра https://function-x.ru/vectors/v27.gif, опущенного из этой точки на прямую (плоскость).

https://function-x.ru/image/vectors5.jpg

Пусть https://function-x.ru/vectors/v01.gif - произвольный вектор (Рис. 5), а https://function-x.ru/vectors/v26.gif и https://function-x.ru/vectors/v28.gif - проекции его начала (точки A) и конца (точки B) на ось l. (Для построения проекции точки A) на прямую проводим через точку A плоскость, перпендикулярную прямой. Пересечение прямой и плоскости определит искомую проекцию.

Составляющей вектора https://function-x.ru/vectors/v29.gif на оси l называется такой вектор https://function-x.ru/vectors/v30.gif, лежащий на этой оси, начало которого совпадает с проекцией начала, а конец - с проекцией конца вектора https://function-x.ru/vectors/v01.gif.

Проекцией вектора https://function-x.ru/vectors/v01.gif на ось l называется число

https://function-x.ru/vectors/v31.gif,

равное длине составляющего вектора на этой оси, взятое со знаком плюс, если направление составляюшей совпадает с направлением оси l, и со знаком минус, если эти направления противоположны.

Основные свойства проекций вектора на ось:

1. Проекции равных векторов на одну и ту же ось равны между собой.

2. При умножении вектора на число его проекция умножается на это же число.

3. Проекция суммы векторов на какую-либо ось равна сумме проекций на эту же ось слагаемых векторов.

4. Проекция вектора на ось равна произведению длины проектируемого вектора на косинус угла между вектором и осью:

https://function-x.ru/vectors/v32.gif

Пример2. Рассчитать проекцию суммы векторов https://function-x.ru/vectors/v227.gif на ось l, если https://function-x.ru/vectors/v228.gif, а углы -

https://function-x.ru/vectors/v229.gif.

Решение. Спроектируем векторы на ось l как определено в теоретической справке выше. Из рис.5а очевидно, что проекция суммы векторов равна сумме проекций векторов. Вычисляем эти проекции:https://function-x.ru/image/vect_proj.jpg

https://function-x.ru/vectors/v230.gif

Находим искомую проекцию суммы векторов:

https://function-x.ru/vectors/v231.gif.


Теорема. Всякий вектор может быть разложен по ортам координатных осей:

https://function-x.ru/chapter4-1/vectors1_clip_image102.gif        (2)https://function-x.ru/image/vect_coord2.jpg

Равенство (2) называется разложением вектора по координатным осям. Коэффициентами этого разложения являются проекции вектора на координатные оси. Таким образом, коэффициентами разложения (2) вектора по координатным осям являются координаты вектора.

После выбора в пространстве определённой системы координат вектор и тройка его координат однозначно определяют друг друга, поэтому вектор может быть записан в форме

https://function-x.ru/chapter4-1/vectors1_clip_image104.gif              (3)

Представления вектора в виде (2) и (3) тождественны.

Условие коллинеарности векторов в координатах

Как мы уже отмечали, векторы называются коллинеарными, если они связаны отношением

https://function-x.ru/vectors/v12.gif.

Пусть даны векторы https://function-x.ru/vectors/v179.gif. Эти векторы коллинеарны, если координаты векторов связаны отношением

https://function-x.ru/vectors/v180.gif,

то есть, координаты векторов пропорциональны.

Пример 3. Даны векторы https://function-x.ru/vectors/v181.gif. Коллинеарны ли эти векторы?

Решение. Выясним соотношение координат данных векторов:

https://function-x.ru/vectors/v182.gif.

Координаты векторов пропорциональны, следовательно, векторы коллинеарны, или, что то же самое, параллельны.

Операции над векторами, заданными в координатной форме

Пусть даны два вектора https://function-x.ru/chapter4-1/vectors1_clip_image086_0002.gifи https://function-x.ru/chapter4-1/vectors1_clip_image130.gif, заданные своими проекциями:

https://function-x.ru/chapter4-1/vectors1_clip_image132.gif

или

https://function-x.ru/chapter4-1/vectors1_clip_image134.gif

https://function-x.ru/chapter4-1/vectors1_clip_image136.gif

или 

https://function-x.ru/chapter4-1/vectors1_clip_image138.gif

Укажем действия над этими векторами.

1.Сложение:

https://function-x.ru/chapter4-1/vectors1_clip_image140.gif

или, что то же

https://function-x.ru/vectors/v43.gif,

т.е. при сложении двух векторов одноимённые координаты складываются.

2.Вычитание:

https://function-x.ru/chapter4-1/vectors1_clip_image142.gif

или, что то же

https://function-x.ru/vectors/v44.gif,

т.е. при вычитании двух векторов одноимённые координаты вычитаются.

3.Умножение вектора на число:

https://function-x.ru/chapter4-1/vectors1_clip_image144.gif

или, что то же

https://function-x.ru/vectors/v42.gif,

т.е. при умножении вектора на число все координаты умножаются на это число.

Пример 4. Даны два вектора:

https://function-x.ru/vectors/v142.gif.

Найти https://function-x.ru/vectors/v143.gif.

Решение:

https://function-x.ru/vectors/v144.gif.

Задания для самостоятельной работы

                                                                                         

1. Найдите расстояние от точки (1,2,-3) до: 1) координатных  плоскостей,

2) осей координат,

3) начала координат.

2. На оси абсцисс найдите точку С(х,0,0), равноудаленную от точек А(1,2,3) и В(-2,1,3).

3. Докажите, что четырехугольник АВСД с вершинами А(1,3,2), В(0,2,4), С(1,1,4) и Д(2,2,2) является параллелограммом.

4. При каких значениях х и у векторы коллинеарны: 1) (2,х,3) и (3,2,у); 2) (у,2,5) и (1,-1,х).

5. При каком значении х векторы перпендикулярны: 1) (2,-1,3) и (1,3,х); 2) (х,-2,1) и (х,2х,4).

6. Даны точки: А(0,1,-1), В(1,-1,2), С(3,1,0) и Д(2,-3,1). Найдите косинус угла между векторами АВ иСД.

7. Даны  точки А(1,0,2) и В(-1,1,1). Найдите:

а) координаты единичного вектора, сонаправленного с вектором АВ,

б) углы, которые вектор АВ составляет с осями координат.

8. Точки А(1,0,2), В(2,1,0), С(1,2,0) являются тремя последовательными вершинами параллелограмма. Найдите сумму координат четвертой вершины.

9. Даны точки А(2,0,1), В(3,2,2) и С(2,3,6). Найдите расстояние от начала координат до точки пересечения медиан треугольника АВС.

10. Даны точки А(1,2,3) и В(0,1,-1). Напишите уравнение плоскости, проходящей через точку А перпендикулярно прямой АВ.

11. Прямая является пересечением плоскостей 2х+3у+z=1 и х+у+z=1.

Укажите какой-нибудь вектор, параллельный этой прямой.

12. В кубе АВСДА1В1С1Д1 точка М лежит на ребре ВВ1, причем ВМ:МВ1=3:2, а точка Н лежит на ребре АД, причем

АН:НД=2:3. Вычислите угол между прямой МН и:

 а) прямой АС1,

б) плоскостью грани ДД1С1С.  



Предварительный просмотр:

Федеральное агентство по образованию
Государственное образовательное учреждение профессионального образования
КАЗАНСКИЙСТРОИТЕЛЬНЫЙ КОЛЛДЕЖ.


ПРАКТИЧЕСКИЕ РАБОТЫ.

I КУРС


Тема
«Комбинаторика»

Составитель преподаватель
математики Садыкова Р.Н.

КАЗАНЬ-2017

Практическая работа №

Элементы комбинаторики

Цель: приобретение базовых знаний в области фундаментальных разделов математики. Повторить и систематизировать знания по данной теме.

Задачи: 

• развитие творческого профессионального мышления;

• познавательная мотивация;

• овладение языком науки, навыки оперирования понятиями;

• овладение умениями и навыками постановки и решения задач;

• углубление теоретической и практической подготовки;

• развитие инициативы и самостоятельности студентов.

Теоретические сведения:

Элементы теории множеств

1. Логические символы

Квантор  - заменяет выражение "для любого", "для произвольного", "для какого бы ни было".

Квантор  - заменяет выражение "существует", "найдется".

Запись  (импликация) означает, что из справедливости высказывания A вытекает справедливость высказывания B. Если, кроме того, из справедливости высказывания B вытекает справедливость A, то записываем . Если  , то высказывание B является необходимым и достаточным условием для того, чтобы выполнялось высказывание A.

Если предложения A и B справедливы одновременно, то записываем  . Если же справедливо хотя бы одно из предложений A или B, то записываем  .

2. Операции над множествами

Математическое понятие множества элементов принимается в качестве интуитивного. Множество задается правилом или признаком, согласно которому определяем, принадлежит ли данный элемент множеству или не принадлежит.

Множество обозначают символом A = {x}, где x - общее наименование элементов множества A. Часто множество записывают в виде A = {a, b, c, ...}, где в фигурных скобках указаны элементы множества A. Будем пользоваться обозначениями:

                  N - множество всех натуральных чисел;

                  Z - множество всех целых чисел;

                  Q - множество всех рациональных чисел;

                  R - множество всех действительных чисел;

                  C - множество всех комплексных чисел;

                  Z0 - множество всех неотрицательных целых чисел.

Запись   означает, что элемент a принадлежит множеству A.

Запись    означает, что элемент a не принадлежит множеству A. Множество B, все элементы которого принадлежат множеству A, называется подмножеством множества A, и при этом записывают

(см. рис. 1).

 

Всегда , так как каждый элемент множества, естественно, принадлежит A. Пустое множество, т. е. множество, не содержащее ни одного элемента, обозначим символом  . Любое множество содержит пустое множество в качестве своего подмножества.

Если  , то A и B называются равными множествами, при этом записывают A = B.

Если  , то множество ℐ элементов множества А , не принадлежащих A, называется дополнением множества A к множеству  (см. рис. 2).

Дополнение множества A к множеству ℐ обозначают символом ; или просто CA, если известно, к какому множеству берется  дополнение. Таким образом,СА=

Если Аℐ , то иногда дополнение множества B к множеству A называют разностью множеств A и B и обозначают A\B (см. рис. 3),      т. е.   

Пусть A и B - подмножества множества ℐ.

Объединением множеств A и B называется множество А⋂В(см. рис. 4)   

 

Аналогично, если   подмножества множества ℐ, то их объединением будет множество

   

Пересечением подмножеств A и B называется множество (см. рис. 5)

   

 

Рисунки 1-6 называются диаграммы Эйлера-Венна.

Аналогично, символом   обозначают пересечение подмножеств          множества ℐ,т.е.    

Два элемента a и b называются упорядоченной парой, если указано, какой из этих элементов первый, какой второй, при этом

    

Упорядоченную пару элементов a и b обозначают символом (a, b).

Аналогично определяется упорядоченная система из n элементов a1, a2, ..., an, которую обозначают символом (a1, a2, ..., an). Элементы a1, a2, ..., an называются координатами упорядоченной системы (a1, a2, ..., an).

Совокупность всевозможных упорядоченных пар (a, b), где, называется произведением множеств A и B и обозначается символом .

Аналогично, символом  обозначают произведение множеств , т. е. совокупность всевозможных упорядоченных систем (a1, a2, ..., an),

где   .

  1. Свойства операций над множествами

Пусть A, B и D - произвольные подмножества множестваℐ . Тогда непосредственно из определений объединения, пересечения и дополнения вытекают следующие предложения:

  1. Закон идемпотентности для объединения и пересечения множеств:

А⋃А=А, А⋂А=А

  1. Закон коммутативности:

А⋃В=В⋃А; А⋂В=В⋂А.

  1. Закон ассоциативности:

А⋃(В⋃D)= (А⋃В)⋃D; А⋂(В⋂D)= (А⋂В)⋂D

  1. Закон дистрибутивности:

А⋂(В⋃D)=(А⋂В)⋃(А⋂D); А⋃(В⋂D)=(А⋃В)⋂(А⋃D)

  1. Закон поглощения:

А⋃(В⋂D)=А, А⋂(А⋃D)=А

  1. Закноны, описывающие свойства пустого и универсального множества по отношению к объединению и пересечению:

А⋃∅=А, А⋂∅=∅, А⋃ℐ=ℐ, А⋂ℐ=А

  1. Законы дополнения: А⋃=, А=
  2. Закон инволютивности дополнения:
  3. Закон Де Моргана: =, =

Истинность каждого тождества проще всего проверяется построением диаграмм Эйлера-Венна.

  1. Выполните самостоятельно:

проверку следующих утверждений диаграммой Эйлера-Венна:

Вариант 1

Вариант 2

Вариант 3


Практическая работа №8

Задачи с применением формул комбинаторики

Цель: Научиться решать задачи на нахождение вероятностей

Теоретические сведения к практической работе

Классическое определение вероятности

Раздел математики, изучающий закономерности случайных событий, называется теорией вероятностей.

Вероятностью Р(А) события А в испытании с равновозможными элементарными исходами называют отношение числа исходов m,  благоприятствующих событию А, к числуn всех исходов испытания.

Пример 1: В партии из 30 миксеров 2 бракованных. Найти вероятность купить исправный миксер.

Аксиомы вероятностей:

Каждому событию А поставлено в соответствие неотрицательное число Р(А), называемое вероятностью события А.

Если события А1, А2 … попарно несовместны, то Р(А12+…)=Р(А1)+Р(А2)+…

Свойства вероятностей:

Вероятность невозможного события равна нулю Р=0.

Вероятность достоверного события равна единице Р=1.

Вероятность произвольного случайного события А заключается между 0 и 1: 0<Р(А)<1.

Пример 2: Из 34 экзаменационных билетов, пронумерованных с помощью чисел от 1 до 34, наудачу извлекается один. Какова вероятность, что номер вытянутого билета есть число, кратное трем.

Решение: Найдем количество чисел от 1 до 34, кратных трем. Это числа 3, 6, 9, 12, 15, 18, 21, 24, 27, 30, 33. Всего таких чисел 11. Таким образом, искомая вероятность

События А и В называются совместными, если они могут одновременно произойти, и несовместными, если при осуществлении одного события не может произойти другое.

События А и В называются независимыми, если вероятность наступления одного события не зависит от того, произошло другое событие или нет.

Вероятность суммы двух совместных событий равна сумме вероятностей слагаемых без вероятности произведения: Р(А+В)=Р(А)+Р(В)-Р(АВ)

Пример 3: Вероятность поражения одной мишени – 0,7, а другой – 0,8. Какова вероятность, что будет поражена хотя бы одна мишень, если по ним стреляют независимо друг от друга.

Решение: Т.к. события совместны, то

Вероятность суммы двух несовместных событий равна сумме вероятностей слагаемых: Р(А+В)=Р(А)+Р(В).

Р(А)+Р()=1

Условная вероятность – вероятность одного события, при условии, что другое событие уже произошло.

Вероятность произведения событий А и В равна произведению вероятности одного из них на условную вероятность другого: Р(АВ)=Р(А)∙Р(А/В) или Р(ВА)=Р(А)∙Р(В/А)

Вероятность произведения двух независимых событий А и В равна произведению вероятностей сомножителей: Р(АВ)=Р(А)∙Р(В).

        Пример 4: В двух коробках лежат ручки разного цвета. В первой коробке – 4 красных и 6 черных, во второй – 3 красных, 5 синих и 2 черных. Из обеих коробок вынимают по одной ручки. Найти вероятность, что обе ручки красные.

Решение: Найдем вероятности вытащить красную ручку из каждой коробки

Тогда вероятность того, что обе ручки красные:  

Полная вероятность. Формула Байеса

Если событие А может произойти только при выполнении одного из событий Н1, Н2, …, которые образуют полную группу несовместных событий, то вероятность события А вычисляется по формуле

Эта формула называется формулой полной вероятности.

Если выполняются все условия, имеющие место для формулы полной вероятности, и , то выполняется равенство, называемое формулой Байеса:

Пример 1: В первой партии 20 ламп, во второй – 30 ламп и в третьей – 50 ламп. Вероятности того, что проработает заданное время, равна для первой партии 0,7, для второй – 0,8 и для третьей партии – 0,9. Какова вероятность того, что наудачу взятая лампа проработает заданное время? Найти вероятность, что эта лампа принадлежит первой партии?

Решение: Пусть событие А – наудачу взятая лампа проработает заданное время.

Тогда, пусть Н1 – лампа из первой партии, Н2 – лампа из второй партии и Н3 – лампа из третьей партии. Тогда событие А/Н1 – лампа из первой партии проработает заданное время, А/Н2 – лампа из второй партии проработает заданное время и А/Н3 – лампа из третьей партии проработает заданное время. Найдем вероятности

Теперь, используя формулу Байеса найдем вероятность того, что эта лампа принадлежит первой партии

 

Пример 2: Имеются 3 одинаковые урны. В первой урне находятся 5 белых и 7 черных шаров, во второй – только белые и в третьей – только черные. Наугад выбираются урна и из нее извлекается один шар. Какова вероятность, что этот шар белый?

Решение: Пусть событие А – извлекается белый шар.

Тогда, пусть Н1 – шар из первой урны, Н2 – шар из второй урны и Н3 – шар из третьей урны. Тогда событие А/Н1 – белый шар из первой урны, А/Н2 – белый шар из второй урны и А/Н3 – белый шар из третьей урны. Найдем вероятности

Формула Бернулли

  1. Вероятность того, что событие А наступит ровно m раз при проведении n независимых испытаний, каждый из которых имеет ровно два исхода вычисляется по формуле Бернулли

Пример 1: Вероятность выигрыша по одному лотерейному билету равна 0,2. Найти вероятность, что из 6 приобретенных билетов 2 окажутся выигрышными.

Решение:

 

  1. Вероятность наступления события А хотя бы один раз при проведении n независимых испытаний, удовлетворяющих схеме Бернулли, равна

Пример 2: Прибор состоит из шести элементов, работающих независимо друг от друга. Вероятность безотказной работы каждого элемента за определенное время равна 0,6. Для безотказной работы прибора необходимо, чтобы хотя бы один элемент был исправен. Какова вероятность, что за данное время прибор будет работать безотказно?

Решение:

 

  1. Вероятность наступления события А хотя бы один раз при проведении n независимых испытаний, удовлетворяющих схеме Бернулли, наступит не менее m1 и не более m2 раз вычисляется по формуле

Пример 3: Найти вероятность осуществления от двух до четырех разговоров по телефону при наблюдении пяти независимых вызовов, если вероятность того, что разговор состоится, равна 0,7.

Решение:

  1. Наивероятнейшее значение m0 числа наступления события А при проведении n повторных независимых испытаний, удовлетворяющих схеме Бернулли, вычисляется по формуле

Пример 4: Магазин получил 50 деталей. Вероятность наличия нестандартной детали в партии равна 0,05. Найти наиболее вероятное число нестандартных деталей в партии.

Решение:

Дискретная случайная величина и ее числовые характеристики

Случайная величина Х – это числовая функция , определенная на пространстве элементарных событий. Случайные величины, имеющие счетные множества возможных значений, называются дискретными. Дискретная случайная величина определена, если известны все ее значения и соответствующие им вероятности. Соотношение между возможными значениями случайной величины и соответствующими им вероятностями называют распределением вероятностей случайной величины. Для дискретной случайной величины это соответствие может быть записано в виде таблицы:

xi

x1

x2

xn

pi

p1

p2

pn

Математическим ожиданием (средним значением) дискретной случайной величины Х называют сумму произведений всех ее возможных значений на соответствующие им вероятности

Дисперсией дискретной случайной величины Х называют математическое ожидание квадрата отклонения случайной величины от ее математического ожидания . Дисперсия дискретной случайной величины вычисляется по формулам:

Средним квадратичным отклонением дискретной случайной величины называют корень квадратный из дисперсии .

Если случайная величина Х имеет биномиальное распределение вероятностей, то

Пример 1: Случайная величина Х задана таблицей распределения вероятностей. Найти М(Х), D(Х), σ(Х).

хi

2

5

8

9

рi

0,1

0,4

0,3

0,2

Решение:

 

Пример 2: Найти математическое ожидание и дисперсию числа лотерейных билетов, на которые выпадут выигрыши, если приобретено 100 билетов, а вероятность выигрыша на каждый билет равна 0,05.

Решение:

Содержание практической работы

Задание 1. Используя классическое определение вероятности события, решить следующие задачи:

1.  В коробке 4 красных, 5 зеленых, 8 желтых, 7 белых и 1 черный шар. Найти вероятность вытащить: красный шар; синий шар; белый шар; цветной шар; или зеленый или белый шар; не красный шар; шар одного из цветов светофора.

2.  В семье – двое детей. Какова вероятность, что старший ребенок – девочка, если известно, что в семье есть дети обоего пола?

3. Мастер, имея 10 деталей, из которых 4 – нестандартных, проверяет детали одну за другой, пока ему не попадется стандартная. Какова вероятность, что он проверит ровно две детали?

4. В одном ящике 3 белых и 7 черных шаров, в другом ящике – 6 белых и 8 черных шара. Найти вероятность того, что хотя бы из одного ящика будет вынут белый шар, если из каждого ящика вынуто по одному шару.

5. Издательство отправило газеты в три почтовых отделения. Вероятность своевременной доставки газет в первое отделение равна 0,9, во второе - 0,7, в третье - 0,85. Найти вероятность следующих событий:

а) только одно отделение получит газеты вовремя;

б) хотя бы одно отделение получит газеты с опозданием.

6. В первой урне находятся 12 белых и 4 черных шаров, а во второй 5 белых и 10 черных шаров. Из каждой урны вынули по шару. Какова вероятность того, что оба шара окажутся черными? Какова вероятность, что оба шара окажутся белыми?

7. В партии из 25 деталей находятся 8 бракованных. Вынимают из партии наудачу две детали. Определить, какова вероятность того, что обе детали окажутся бракованными.

8. Подброшены две игральные кости. Найти вероятность события A того, что выпадет хотя бы одна шестерка.

9. Найти вероятность, что при бросании игральной кости выпадет число, большее 4.

10. Найти вероятность, что при бросании игральной кости выпадет число, не меньшее 2 и не большее 5.

Задание 2. Используя формулы полной вероятности и Байеса, решить следующие задачи:

1. Имеются 2 одинаковые урны. В первой урне находятся 7 белых и 3 черных шаров, во второй – 6 белых и 4 черных. Наугад выбираются урна и из нее извлекается один шар. Выбранный шар оказался черным. Какова вероятность, что этот шар из 2 урны?

2. Детали, изготовляемые цехом завода, попадают для проверки их на стандартность к одному из двух контролеров. Вероятность того, что деталь попадет к первому контролеру =0,5, ко второму =0,6. Вероятность того, что годная деталь будет признана стандартной первым контролером =0,94, а вторым =0,92. Годная деталь при проверке была признана стандартной. Найти вероятность того, что эту деталь проверил первый контролер.

3. Имеется два набора деталей. Вероятность того, что деталь первого набора стандартная равна 0,9, а второго – 0,8. Найти вероятность того, что взятая наудачу деталь – стандартная.

4. Имеются 3 одинаковые урны. В первой урне находятся 6 синих и 4 черных шаров, во второй – только синие и в третьей – только черные. Наугад выбираются урна и из нее извлекается один шар. Какова вероятность, что этот шар синий?

5. Имеются 2 одинаковые урны. В первой урне находятся 7 белых и 3 черных шаров, во второй – 6 белых и 4 черных. Наугад выбираются урна и из нее извлекается один шар. Выбранный шар оказался черным. Какова вероятность, что этот шар из 1 урны?

Задание 3. Используя формулу Бернулли, решить следующие задачи:

1. Вероятность того, что расход электроэнергии на продолжении одних суток не превысит установленной нормы равна 0,75. Найти вероятность того, что в ближайшие 6 суток расход электроэнергии в течение 4 суток не превысит нормы.

2. Найти вероятность осуществления от одного до трех разговоров по телефону при наблюдении шести независимых вызовов, если вероятность того, что разговор состоится, равна 0,6.

3. Прибор состоит из пяти элементов, включенных в цепь параллельно и работающих независимо друг от друга. Вероятность безотказной работы каждого элемента за время Т равна 0,5. Для безаварийной работы прибора достаточно, чтобы хотя бы один элемент был исправен. Какова вероятность того, что за время Т прибор будет работать безотказно?

4. Вероятность выигрыша по одному лотерейному билету =0,3. Какова вероятность того, что из семи приобретенных билетов три билета окажутся выигрышными?

5. Магазин получил 40 деталей. Вероятность наличия нестандартной детали в партии равна 0,04. Найти наиболее вероятное число нестандартных деталей в этой партии.

6. Вероятность изготовления на автоматическом станке стандартной детали равна 0,8. Найдя вероятности возможного числа появления бракованных деталей среди 5 отобранных, найти наивероятнейшее число появления бракованных деталей из 5 отобранных, указав его вероятность.

7. Сколько раз необходимо подбросить игральную кость, чтобы наивероятнейшее выпадение тройки было равно 10?

8. Для данного участника игры вероятность набросить кольцо на колышек =0,3. Какова вероятность того, что при шести бросках 3 кольца окажутся на колышке?

9. На самолете имеются 4 одинаковых двигателя. Вероятность нормальной работы каждого двигателя в полете равна р. Найти вероятность того, что в полете могут возникнуть неполадки в одном двигателе.

10. Вероятность отказа каждого прибора при испытании равна 0,4. Что вероятнее ожидать: отказ двух приборов при испытании четырех или отказ трех приборов при испытании шести, если приборы испытываются независимо друг от друга?

11. Вероятность того, что на некотором предприятии расход электроэнергии не превысит суточной нормы равна 0,8. Какова вероятность того, что в течение пяти рабочих дней из семи перерасхода электроэнергии не будет?

Задание 4. Найти числовые характеристики дискретных случайных величин:

1. Найти математическое ожидание случайной величины Х, зная закон ее распределения:

хi

3

5

2

рi

0,1

0,6

0,3

2. Вероятность попадания в цель при стрельбе из орудия 0,6. Найти математическое ожидание общего числа попаданий, если будет произведено 10 выстрелов.

3. Найти дисперсию случайной величины Х, которая задана следующим законом распределения:

хi

1

2

5

рi

0,3

0,5

0,2

4.Найти дисперсию случайной величины Х, которая задана следующим законом распределения:

хi

2

3

5

рi

0,1

0,6

0,3

5. Производится 10 независимых испытаний, в каждом из которых вероятность появления события равна 0,6. Найти дисперсию случайной величины Х – числа появления события в этих испытаниях.




Предварительный просмотр:

Федеральное агентство по образованию
Государственное образовательное учреждение профессионального образования
КАЗАНСКИЙСТРОИТЕЛЬНЫЙ КОЛЛДЕЖ.


ПРАКТИЧЕСКИЕ РАБОТЫ.

I КУРС


     Тема
«Элементы теории вероятности и математической статистики»

Составитель преподаватель
математики Садыкова Р.Н.

КАЗАНЬ-2017

Практическая работа №9

Вероятность события

Цель: приобретение базовых знаний в области фундаментальных разделов математики. Повторить и систематизировать знания по данной теме.

Задачи: 

• развитие творческого профессионального мышления;

• познавательная мотивация;

• овладение языком науки, навыки оперирования понятиями;

• овладение умениями и навыками постановки и решения задач;

• углубление теоретической и практической подготовки;

• развитие инициативы и самостоятельности студентов.

Теоретические сведения:

1.  Приведите примеры: 1) достоверных событий; 2) невозможных событий; 3) случайных событий.

2.  Что вероятнее — появление герба при бросании монеты или появление нечетного числа очков при бросании игральной кости?

3.  Что вероятнее при бросании двух монет — выпадение двух цифр или цифры и герба?

4.  Что вероятнее получить при делении домино между 4 игроками — все «дубли» или же все кости с «шестерками»?

5.  Проведите следующий эксперимент: бросьте 50 раз две игральные кости и запишите сумму для каждого броска. Какая сумма появилась чаще всего? Какая реже всего? Какое число очков появилось чаще: 3 или 12?

6.  Из мешка с 33 жетонами, на которых написаны русские буквы, вытаскивают один за другим 4 жетона. Сколько раз, по вашему мнению, нужно повторить этот эксперимент, чтобы из этих букв получилось слово «барс»? Во сколько раз будет меньше число необходимых экспериментов, если 4 жетона вытаскивают сразу (т. е. порядок их появления несуществен)?

7.  Что вероятнее — угадать 6 номеров из 49 или 5 номеров из 36?

8.  При 10 бросаниях правильной монеты выпадал герб. Что вероятнее при следующем броске — выпадение цифры или герба?

2. Опыт, испытание. Основным понятием, с которым мы будем иметь дело в дальнейшем, является понятие опыта, или испытания. Этому понятию нельзя дать математическое определение, однако ясно, что значат слова «подбросим монету и посмотрим, упала она вверх гербом или цифрой» или «включить электрическую лампочку и поглядеть, через какое время она перегорит». Для нас будет существенно лишь то, что данное испытание может иметь различные исходы. При этом для простоты будем рассматривать лишь случаи, когда множество  этих исходов конечно и равно n. С каждым опытом можно связать различные множества исходов. Важно лишь то, что при каждом испытании происходит один и только один исход.

Пример 1. При бросании игральной кости возможны следующие  исходы:

1)   А1, А2, А3, А4, А5, А6  это означает выпадение  очков от 1 до 6 включительно;

2)   В1 — выпадение нечетного числа очков, а В2 — выпадение четного числа очков;

Пример 2. При бросании монеты возможны исходы А1 – выпал герб, А2 – выпала «решка»

Пример 3. Произведен выстрел по мишени: А1 – попадание, А2 – промах.

Введем следующее определение:

Определение . Событием при данном испытании называется любое подмножество X множества U исходов.

В дальнейшем, говоря об исходах, из которых состоит событие X, мы будем говорить, что они благоприятствуют этому событию. Про остальные же исходы будем говорить, что они не благоприятствуют событию X.

Определение 2. Вероятностью события X называют сумму вероятностей исходов, благоприятствующих этому событию.

Пример 1. Бросают игральную кость, событие А – выпадение четного числа очков. Ему благоприятствуют случаи А2, А4, А6, т.о. 3 исхода из 6-ти возможных.

Пример 2. Бросают монету, событие В – выпадение герба, ему благоприятствует один исход из двух возможных.

Если испытание может привести к одному и только одному из n различных равновозможных исходов и если m из этих исходов благоприятствуют появлению события А определяется формулой

Р(А)=m/n

Это классическое определение вероятности.

Основные свойства:

  1. Вероятность любого события заключена между 0 и единицей:   0≤ Р(А)≤ 1
  2. Вероятность достоверного события U, т.е. такое событии обязательно произойдет при испытании: Р(U)=1
  3. Вероятность испытание невозможного события V равна нулю: Р(V)=0
  4. Сумма вепроятностей двух противоположных событий А и Ā, т.е. таких событий, что появление одного из них исключает появление другого, равна единице:

                          Р(А)+Р(Ā)=1

Пример 1.

Из урны, в которой находится 4 белых, 9 черных, 7 красных шаров. Наудачу вынимают один шар. Какова вероятность, что этот шар белый?

Решение:

Элементарным исходом является извлечение любого шара. Число таких исходов равно числу шаров: 4+9+7=20, т.е. n=20. Событие А – извлечение белого шара, ему благоприятствуют 4 исхода, т.к. белых шаров 4, значит m=4, поэтому по формуле Р(А)=m/n находим: Р(А)=4/20=1/5=20%

Пример 2. Задача о выборке.

В партии из S деталей имеются Т нестандартных. Определить вероятность того, что среди выбранных наудачу p деталей нестандартными окажутся ровно t деталей.

Решение:

Элементарным исходом является выборка любых p изделий из общего числа S. Число таких исходом равно числу сочетаний из S по p, т.е.n=           

Интересующее нас событие А – это извлечение p деталей, из которых t нестандартные. Следовательно, благоприятными для А являются такие группы по p изделий, в которых p-t изделий – качественные, а t – нестандартные.

Число таких групп

              m=·, где , причем события из группы стандартных комбинируются из группы нестандартных, тогда

Р(А)=

2. ТЕОРЕМЫ СЛОЖЕНИЯ И УМНОЖЕНИЯ ВЕРОЯТНОСТЕЙ

Суммой нескольких событий называется событие, состоящее в появлении хотя бы одного из этих событий.

Произведением нескольких событий называется событие, состоящее в совместном осуществлении всех этих событий.

Теорема сложения вероятностей.

 Если события Ах, А2, ... , А п несовместны, т. е. никакие два из них не могут осуществиться вместе, то

P(А1+ А2+ ...+ Ап) = Р (A1) + Р(А2) + ... + Р(Аn)      (1)

Вероятность события А, вычисленная в предположении, что произошло событие В, называется условной вероятностью события А при условии В {обозначается Р (А/В).

Теорема умножения вероятностей.

 Вероятность произведения нескольких независимых событий равна произведению вероятности этих событий:

Р(А1А2  ... Аn) = = Р (A1)·P (А2) ·Р(А3)   ... Р (Аn )      (2)

(события А1, А2, ..., Ап независимы, т. е. осуществление любого числа из них не меняет вероятностей осуществления остальных).

Пример 3.   В партии из 50 изделий содержится пять бракованных. Какова вероятность того, что из выбранных наудачу 30 изделий не более одного бракованного?

Решение. Пусть А — событие, состоящее в том, что 30 изделий выборки — качественные, В — в рассматриваемой выборке из 30 изделий только одно бракованное, С— не более одного бракованного. Тогда, очевидно, С = А + В. Так как события А и В несовместны, то по формуле (1) имеем

Р(С) = Р(А) + Р(В).

Найдем вероятности событий А и В:

Р(А)=               ≈ 0,007                               Р(В)=           ≈0,065

Отсюда  Р(С)≈ 0,072

Пример 4.   Два станка работают независимо друг от друга. Вероятность бесперебойной работы первого станка в течение некоторого времени t равна p1= 0,9, второго — р2= 0,8. Какова вероятность бесперебойной работы обоих станков в течение указанного промежутка времени?

Решение. Рассмотрим следующие события: A1 и А2— бесперебойная работа соответственно первого и второго станков в течение времени t; A — бесперебойная работа обоих станков в течение указанного времени. Тогда событие А есть совмещение событий А1 и А2, т. е. А  = А1·A2. Так как события At и А2 независимы (станки работают независимо друг от друга), то по формуле (3) получим

Р(А) = Р(А1)·Р(А2) = 0,9·0,8 =0,72.

Пример 5.   В примере 3 определить вероятность бесперебойной работы хотя бы одного из двух станков в течение времени   t   (событие   В).

Решение. Первый способ. Рассмотрим противоположное событие В, означающее простой обоих станков в течение времени /. Очевидно, что событие В есть совмещение событий A1 и А2 простоев первого и второго станков, т. е. В = Ā1 ·Ā2. Так как события Ā1 и Ā2? независимы,   то

Р(В) = Р (Ā1) .Р(Ā2) = ( 1- Р(А1))· ( 1- Р(А2)) = = 0,1·0,2 = 0,02.

Отсюда

Р{В) = 1 — Р(В) = 0,98.

Второй способ. Событие В происходит в том случае, когда имеет место одно из следующих трех несовместных событий: либо A1·Ā2—совмещение событий A1 4 и Ā2 (первый станок работает, второй — не работает), либо Ā1·А2— совмещение событий   Ā1  и А2 (первый станок не работает, второй — работает), либо А1А2— совмещение событий А1 и Л2 (оба станка работают), т. е.

В = А1 Ā21А2+ A1A2.

По формуле (1) получим

Р(В) = Р(А1    Ā2) + P(Ā1A2) + P(A1A2).

В силу того, что события A1 и А2, а следовательно, Ā1 и Ā2,  независимы, имеем

Р(В)=Р(А1)·Р(Ā2)+Р(Ā1).Р(А2) + Р(А1)·Р(А2) = = Р(А1)[1-Р(А2)]+[1-Р(А1)]Р(А2) + Р(А1).Р(А2)=0,98.

Формула полной вероятности. Формула Байеса

Если   с  некоторым  опытом  связано   п   исключающих  друг друга событий (гипотез)Н1 , Н2, ..., Нn и если событие А может осуществиться только при одной из этих гипотез, то вероятность события А вычисляется по формуле полной вероятности:

 Р(А) = P(H1)P(A/H1) + Р(А2)Р(А/Н2) +   ... + Р(Нп)Р(А/Нп).

После проведения опыта, в результате которого осуществилось событие А, вероятности гипотез Нi можно переоценить по формуле   Байеса:

Р (Нi / А) =   Р(Нi) · Р( А / Нi )

                                    Р (А)

Пример 5. Имеется три урны с шарами. В первой урне 4 белых и 5 черных, во второй — 5 белых и 4 черных, в третьей — 6 белых шаров. Некто выбирает наугад одну из урн и вынимает из нее шар. Найти вероятность того, что: а) этот шар окажется белым; б) белый шар вынут из второй урны.

Решение, а) Пусть А — событие, означающее, что извлечен белый шар. Рассмотрим три гипотезы:

H1— выбрана первая урна; Н2— выбрана вторая урна ; Нз—     третья .Так как урна, из которой извлекают шар, выбирается наугад, то

Р(Н1) = Р(Н2) = Р(Нз) = ⅓

Условные вероятности события А соответственно равны:

P(A/Н1) = 4/9  (вероятность   извлечения   белого   шара    из первой урны),

Р(А/Н2) = 5/9 (вероятность   извлечения   белого   шара  из второй урны),

Р(А/Н3) = 1    (вероятность    извлечения    белого    шара   из третьей урны).

а) Отсюда по формуле полной вероятности получим

Р(А)=1/3×4/9 + 1/3×5/9 + 1/3×1 = 2/3

б) Для определения вероятности того, что белый шар извлечен из второй урны, воспользуемся формулой Байеса:

Р{Н2 /А) = Р(Н2)Р(А/Н2) ‗  1/3×5/9         

                       P(A)                 2/3

Схема повторных испытаний. Формула Бернулли

Если при одних и тех же условиях определенный опыт повторяется п раз и если вероятность появления некоторого события А в каждом опыте равна р, то вероятность того, что событие А в серии из п опытов произойдет ровно k раз, находится по формуле Бернулли:

Рn (k) = С   рk  q n-k  , где q=1- р


Выполните самостоятельно:

Вариант 1

№1.

 В группе 20 студентов, среди них 14 юношей. Найти вероятность того, что среди наудачу выбранных 6-ти студентов будут 3 девушки и 3 юноши.

№2.

 Имеются 4 коробки с шарами.

1-я: 4 синих и 5 красных;

2-я: 5 синих и 4 красных;

 3-я: 7 красных;

4-я: 12 синих.

Наудачу берут шар. Он красный. Найти вероятность того, что он из 2-й коробки.

№3

Двум студентам предложена задача. Вероятность того, что её решит 1-й студент равна 0,72, что решит 2-й – 0,65. Найти вероятность того, что задачу решат оба студента; что решит только один?

Вариант 2

№1

Имеются 23 детали и среди них 19 стандартные. Случайным образом выбирают сразу 6. Какова вероятность, что среди выбранных ровно 5 стандартных?

№2

В цехе продукция производится на 3-х станках:

1-й станок 45% всей продукции, из них брак 5%;

2-й станок 35% всей продукции, из них брак 10%;

3-й станок 20% всей продукции, из них брак  2%.

Найти вероятность, что наудачу взятая деталь из всех произведенных стандартная. Какова вероятность, что она была произведена на 1-м станке?

№3

Два стрелка независимо друг от друга производят выстрел по мишени. Вероятность попадания 1-м -  

0,8, 2-м – 0,9. Какова вероятность, что после одного выстрела в мишени будет только одна пробоина?

Вариант 3

№1

В урне лежат шары: 7 белых, 4 черных и 9 красных. Наудачу вынимают сразу два шара. Какова вероятность, что они окажутся одного цвета?

№2

В автоколонне 12 машин. Вероятность выхода на линию каждой машины – 0,8. Найти вероятность, что работа автоколонны будет осуществляться без сбоев, если для этого требуется, чтоб не менее 10 машин вышли на линию?

№3

Цех производит продукцию на 2-х станках:

70% изготавливается на 1-м станке, среди них 12% составляют бракованные детали, остальные детали производятся на втором станке, среди них 15% бракованные. Какова вероятность, что наудачу взятая деталь окажется бракованной? Какая вероятность, что бракованная деталь произведена на 2-м станке?

Вариант 4

№1

Три стрелка стреляют независимо друг от друга по цели. Вероятность попадания 1-м -0,8; 2-м – 0,75; 3-м – 0,7. Найти вероятность того, что будет:

1) хотя бы одно попадание;

2) ровно одно попадание;

если произведен один выстрел каждым.

№2

В магазин поступают часы, выпускаемые на 3-х заводах. Первый завод поставляет 40%, второй – 45%, третий – 15%. В продукции первого завод 20% часов спешат, второго завода – 30% часов спешат, третьего – 10% спешат. Найти вероятность того, что купленные часы спешат?

№3

Какова вероятность, что при десяти бросках игральной кости пять очков выпадут ровно 3 раза?

Вариант 5

№1

В мастерской работают 6 моторов. Для каждого мотора вероятность перегрева к обеденному перерыву равна 0,8. Найти вероятность того, что к обеденному перерыву перегреются:

1) ровно 4 мотора;

2) перегреются все моторы?

№2

Детали на сборку попадают из трёх автоматов. Известно, что первый автомат дает 3% брака, второй – 2% брака, третий – 4% брака. Найти вероятность попадания на сборку бракованной детали, если 1-й автомат произвел 1000 деталей, 2-й – 2000 деталей и 3-й – 2500 деталей. Какова вероятность, что бракованная деталь произведена на 2-м автомате?

№3

Из 3000 лотерейных билетов выигрышными являются 12. Какова вероятность, что из наудачу взятых   15 билетов хоть один будет с выигрышем?

Вариант 6

№1

В белом ящике 12 красных и  6 синих шаров, в желтом ящике 15 красных и 10 синих шаров. Наудачу из некоторого ящика выбирают шар. Какая вероятность, что он красный? Какова вероятность, что красный шар  вынут из белого ящика?

№2

По самолету противника производят три выстрела. Вероятность попадания при 1-м выстреле-0,5, при 2-м – 0,6, при 3-м – 0.8. Вероятность сбить самолет при условии попадания при 1-м выстреле – 0,3, при 2-м – 0,6 и при 3-м – 0,9. Найти вероятность того, что самолет будет сбит. Какова вероятность, что он будет сбит при 1-м выстреле?

№3

Два студента решают задачу независимо друг от друга. Вероятность того, что решит 1-й – 0,7, что решит 2-й – 0,8. Найти вероятность того, что:

а) решат оба;

б) решит только один?



Предварительный просмотр:

МИНИСТЕРСТВО ОБРАЗОВАНИЯ И НАУКИ РФ.
Федеральное агентство по образованию
Государственное образовательное учреждение профессионального образования
КАЗАНСКИЙСТРОИТЕЛЬНЫЙ КОЛЛДЕЖ.


ДОМАШНИЕ ПРАКТИЧЕСКИЕ РАБОТЫ.(ДПР)

II КУРС


Тема
« Теория линейных уравнений»

Составитель преподаватель
математики Садыкова Р.Н.

КАЗАНЬ-2020

ДПР Р.1.1

Обучающая часть работы.

 

Пример 1.Решение системы методом Крамера:

Решить систему линейных уравнений методом Крамера и методом Гаусса: ... -  Учеба и наука - Математика

https://www.liveexpert.ru/public/uploads/2016/12/20/a84a529c82cd0044be37e3e661c3b508.gif

Приступим к нахождению неизвестной переменной x1. Для этого умножим обе части первого уравнения системы на А1 1 , обе части второго уравнения – на А2 1 , и так далее, обе части n-ого уравнения – на Аn 1 (то есть, уравнения системы умножаем на соответствующие алгебраические дополнения первого столбца матрицы А. Сложим все левые части уравнения системы, сгруппировав слагаемые при неизвестных переменных x1, x2, …, xn, и приравняем эту сумму к сумме всех правых частей уравнений

Пример 2.Решение системы методом Гаусса:

https://www.liveexpert.ru/public/uploads/2016/12/20/4e3fd7ff3e07ea63c59589189b0f6e41.gif

https://www.liveexpert.ru/public/uploads/2016/12/20/eab846c83264f0115a339b9b4430c0c4.gif

Далее первую строчку переписываем без изменений, а во второй мы на месте a21a должны сделать ноль, Для этого надо умножить первую строку на -2 и сложить её со второй, а результат записать на место второй. Потом надо сделать ноль на месте элемента a31, для этого мы первую строку умножим и додадим до третьей, а результат нужно записать в третью строку. Потом первую и вторую строки переписываем без изменений, а на месте элемента a32 надо сделать ноль, для этого помножим вторую строку на 4, третью на -7 и додадим их, результат записываем в третью строку. Вот мы и преобразовали матрицу к треугольной. По этой матрице запишем систему уравнений.

Пример 3.Решение системы обратным матричным методом:

image014.gif

Матричное уравнение запишется в виде: Y·A = B.Вычислим определитель матрицы А:
∆ = 3*(1*0 - 3*4) - 2*(0*0 - 3*5) + -1*(0*4 - 1*5) = -1.Определитель матрицы А равен detA=-1
Так как A невырожденная матрица, то существует обратная матрица A-1. Умножим справа обе части уравнения на A-1: X·A·A-1 = B·A-1, откуда находим, что X = B·A-1

Домашняя практическая работа для самостоятельного решения:

№ 1. Решить систему линейных уравнений(методом Гаусса, Крамера, матричный).

  1.        2)

 3)             4)

№ 2. Вычислить определитель с  помощью «правила треугольника».

А)   ;    Б)

№ 3. Найти обратную матрицу для матрицы:

А)    ; Б)

 ДПР Р.1.2

Обучающая часть работы.

Пример 1.Решение системы методом Крамера:

 http://botaniks.ru/imgprimerizadach/reshenie_3mya_metodami_gaysa_kramera_obratnoi_matrichi/3.jpg

           

Вычислим определитель, для этого заменим первый столбец в основной матрице на столбец свободных членов. Аналогично, заменяя второй столбец основной матрицы. Далее по формуле Крамера находим неизвестные переменные

Пример 2.Решение системы методом Гаусса:

http://botaniks.ru/imgprimerizadach/reshenie_3mya_metodami_gaysa_kramera_obratnoi_matrichi/2.jpg

Коэффициенты необходимо представить в виде таблицы. С правой стороны в отдельном столбце записаны свободные члены. Данный блок отделен для удобства решения. Матрицу со столбцом со свободными членами называют расширенной. Затем основная матрица с коэффициентами приводится к верхней треугольной форме. Данное действие является ключевым моментом при решении системы уравнений с помощью метода Гаусса. Затем нужно выполнить преобразования. В результате матрица должна приобрести треугольный вид. Для этого следует умножить первую строку на (3) и умножить вторую строку на (-1). В результате суммирования второй и первой строк получается следующее.

Пример 3.Решение системы обратным матричным методом :

http://botaniks.ru/imgprimerizadach/reshenie_3mya_metodami_gaysa_kramera_obratnoi_matrichi/4.jpg

Обозначим отдельно как A матрицу коэффициентов при неизвестных и как B матрицу неизвестных и матрицу свободных членов. То есть, для нахождения решений системы нужно обе части уравнения умножить на матрицу, обратную матрице коэффициентов при неизвестных А-1 и приравнять соответствующие элементы полученных матриц.

Домашняя практическая работа для самостоятельного решения:

№1. Решить систему линейных уравнений (методом Гаусса,Крамера,матричный).

1)                                           2)

3)                                        4)

   

 ДПР Р.1.3

Теоретические вопросы.

1.Матрица.

2.Система линейного уравнений.

3.Транспонирование матрицы.

4.Определитель матрицы.

5.Диагональная матрица.

6.Единичная матрица.

7.Матрица канонического вида.

8.Метод Гаусса.

Обучающая часть работы.

Пример 1.Решение системы методом Крамера:

 http://botaniks.ru/imgprimerizadach/reshenie_3mya_metodami_gaysa_kramera_obratnoi_matrichi/3.jpg

           

Вычислим определитель, для этого заменим первый столбец в основной матрице на столбец свободных членов. Аналогично, заменяя второй столбец основной матрицы. Далее по формуле Крамера находим неизвестные переменные

Пример 2.Решение системы методом Гаусса:

https://www.liveexpert.ru/public/uploads/2016/12/20/4e3fd7ff3e07ea63c59589189b0f6e41.gif

https://www.liveexpert.ru/public/uploads/2016/12/20/eab846c83264f0115a339b9b4430c0c4.gif

Далее первую строчку переписываем без изменений, а во второй мы на месте a21a должны сделать ноль, Для этого надо умножить первую строку на -2 и сложить её со второй, а результат записать на место второй. Потом надо сделать ноль на месте элемента a31, для этого мы первую строку умножим и додадим до третьей, а результат нужно записать в третью строку. Потом первую и вторую строки переписываем без изменений, а на месте элемента a32 надо сделать ноль, для этого помножим вторую строку на 4, третью на -7 и додадим их, результат записываем в третью строку. Вот мы и преобразовали матрицу к треугольной. По этой матрице запишем систему уравнений.

Пример 3.Решение системы обратным матричным методом:

image014.gif

Матричное уравнение запишется в виде: Y·A = B.Вычислим определитель матрицы А:
∆ = 3*(1*0 - 3*4) - 2*(0*0 - 3*5) + -1*(0*4 - 1*5) = -1.Определитель матрицы А равен detA=-1
Так как A невырожденная матрица, то существует обратная матрица A-1. Умножим справа обе части уравнения на A-1: X·A·A-1 = B·A-1, откуда находим, что X = B·A-1


Домашняя практическая работа для самостоятельного решения:


№ 1. Решить систему линейных уравнений(методом Гаусса, Крамера, матричный).

  1.                                     2)

3)                                            4)

5)                                           6)

№ 2. Вычислить определитель с  помощью «правила треугольника».

А)  ;   Б)

№ 3. Решить систему линейных уравнений(методом Гаусса, Крамера, матричный).

1)                                           2)

3)                                        4)

    5)                                           6)

№4. Найти матрицу Х , удовлетворяющую уравнению. Сделать проверку.

А) ; Б) ; В)



Предварительный просмотр:

МИНИСТЕРСТВО ОБРАЗОВАНИЯ И НАУКИ РФ.
Федеральное агентство по образованию
Государственное образовательное учреждение профессионального образования
КАЗАНСКИЙСТРОИТЕЛЬНЫЙ КОЛЛДЕЖ.


ДОМАШНИЕ ПРАКТИЧЕСКИЕ РАБОТЫ.(ДПР)

II КУРС


Тема
« Теория пределов»

Составитель преподаватель
математики Садыкова Р.Н.

КАЗАНЬ-2020

 ДПР Р.2.1

Обучающая часть работы.

Пример 1.Вычисление предела.
unnamed.gif

Пример 2 .Вычисление предела.

unnamed (1).gif

Подставить точку х в выражение, следующее после знака предела. Если получается определенное число, либо бесконечность, то предел решен полностью. В противном случае имеем неопределенность: "ноль делить на ноль" или "бесконечность делить на бесконечность" и переходим к следующим пунктам инструкции.Чтобы устранить неопределенность "ноль делить на ноль" нужно разложить числитель и знаменатель на множители. Сократить подобные. Подставить точку х в выражение, стоящее под знаком предела.Если неопределенность "бесконечность делить на бесконечность", тогда выносим и в числителе, и в знаменателе x наибольшей степени. Сокращаем иксы. Подставляем значения икса из под предела в оставшееся выражение.


Домашняя практическая работа для самостоятельного выполнения:

№1. Вычислить предел
1)
(  1) )
2)
    (  2)  )
3)
   (  3)   )

№ 2. Найти точки разрыва функции:
                                         (  )
№ 3. Исследовать непрерывность функции в точке
 в точке х=2                     (  в точке х=5 )

Практическая работа № 2 ДПР Р.2.2

Обучающая часть работы.

Пример 1.Вычисление предела:

lim46.gif

Домашняя практическая работа для самостоятельного выполнения:

№1. Вычислить предел
1)
( 1)  )
2)
       (2)   )
3)
(3) )
4)
         (  4)    )
5)
(   5)  )


 ДПР Р.2.3

Теоретические вопросы.

  1. Теоремы о пределах
  2. Неопределенности при вычислении пределов
  3. Бесконечно малая функция.
  4. Бесконечно большая функция.
  5. Первый замечательный предел.
  6. Второй замечательный предел.
  7. Непрерывность функции в точке.
  8. Непрерывность функции на промежутке.
  9. Точки разрыва.

Обучающая часть работы.

Пример 1.Вычисление предела:

46a143505744bf6e1c97940c4dd5b0a5.png

Домашняя практическая работа для самостоятельного выполнения:

№1. Вычислить предел
1)
                             (  1)  )
2)
                           (  2)   )
3)
                                          (  3)   )
4)
                                        (  4)     )
5)
                                          (   5)  )
№ 2 Доказать , что данная функция непрерывна на всей области определения:

                        (    )



Предварительный просмотр:

МИНИСТЕРСТВО ОБРАЗОВАНИЯ И НАУКИ РФ.

Федеральное агентство по образованию Государственное образовательное учреждение профессионального образования КАЗАНСКИЙСТРОИТЕЛЬНЫЙ КОЛЛДЕЖ.

ДОМАШНИЕ ПРАКТИЧЕСКИЕ РАБОТЫ.

II КУРС

Тема

« Теория дифференциального исчисления»

Составитель преподаватель

 математики Садыкова Р.Н.

КАЗАНЬ-2020

ДПР Р.3.1

Обучающая часть работы.

Пример 1. Найти наибольшее и наименьшее значение функции

1561552003u.jpg

Домашняя практическая работа для самостоятельного выполнения:

№1.Решить.

  1)Какой закон движения тела по оси Ох, если оно начало двигаться из точки М (10;0,8),со скоростью V=10t+0,4t2

2)Написать формулу Тейлора с остаточным членом в форме Лагранжа для функции

f(x)=Inx,x0=1,n=5

 3)Какой закон движения тела по оси Ох, если оно начало двигаться из точки М (2;-2),со скоростью V=2t-t2

4)Найти наибольшее и наименьшее значение функции в области существования:

5)Какой закон движения тела по оси Ох, если оно начало двигаться из точки М (0;2),со скоростью V=2t3+3t

ДПР Р.3.2

Обучающая часть работы.

Пример 1. Найти наибольшее и наименьшее значение функции:

1542083426r.jpg

 Домашняя практическая работа для самостоятельного выполнения:

№1.Решить.

 1)Какой закон движения тела по оси Ох, если оно начало двигаться из точки М (10;1),со скоростью V=10t+t

2)Написать формулу Тейлора с остаточным членом в форме Лагранжа для функции

3)Найти наибольшее и наименьшее значение функции в области существования:

           4)Найти производную функции, указав D(f) и D(f `),

            5)Используя правило Лопиталя-Бернулли, найти предел

ДПР Р.3.3

Теоретические вопросы.

1)Дифференциальное исчисление.

2)Производные.

3)Дифференциал.

4)Теоремы о дифференцируемых функций.

5)Экстремум функции двух переменных.

6)Касательная кривая.

7)Нормальная к кривой.

8)Выпуклая.

9)Вогнутая.

10)Асимптота.

11)Вертикальная асимптота.

12)Наклонная асимптота.

13)Общий план построения графиков и исследования функций.



Предварительный просмотр:

МИНИСТЕРСТВО ОБРАЗОВАНИЯ И НАУКИ РФ.

Федеральное агентство по образованию Государственное образовательное учреждение профессионального образования КАЗАНСКИЙСТРОИТЕЛЬНЫЙ КОЛЛДЕЖ.

ДОМАШНИЕ ПРАКТИЧЕСКИЕ РАБОТЫ.

II КУРС

Тема

 « Теория интегрального исчисления»

Составитель преподаватель

 математики Садыкова Р.Н.

КАЗАНЬ

ДПР Р.4.1

Обучающая часть работы.

1.Вычислить неопределенный и определенный интеграл.

Пример нахождения неопределенного интеграла.

http://www.mathprofi.ru/f/integraly_primery_reshenij_clip_image054.gif

Решение:

http://www.mathprofi.ru/f/integraly_primery_reshenij_clip_image056.gif

Пример нахождения определенного интеграла.

http://www.mathprofi.ru/f/opredelennye_integraly_primery_reshenij_clip_image052.gif

Решение:

http://www.mathprofi.ru/f/opredelennye_integraly_primery_reshenij_clip_image054.gif

(1) Выносим константу за знак интеграла. (2) Интегрируем по таблице с помощью самой популярной формулы http://www.mathprofi.ru/f/opredelennye_integraly_primery_reshenij_clip_image056.gif. Появившуюся константу  целесообразно отделить от  х3 и вынести за скобку. Делать это не обязательно, но желательно – зачем лишние вычисления. (3) Используем формулу Ньютона Лейбница http://www.mathprofi.ru/f/opredelennye_integraly_primery_reshenij_clip_image062.gif.  Сначала подставляем в х3 верхний предел, затем – нижний предел. Проводим дальнейшие вычисления и получаем окончательный ответ.

Задачи для самостоятельного  выполнения

№2.Вычислить неопределенный и определенный интеграл.

1)                                                           5)

2)                                                           6)                                                   

3)                                                7)

4)                                                           8)

ДПР   Р.4.2

Обучающая часть работы.

Вычислить неопределенный и определенный интеграл.

Пример нахождения определенного интеграла.

http://www.mathprofi.ru/f/opredelennye_integraly_primery_reshenij_clip_image067.gif

Решение:

http://www.mathprofi.ru/f/opredelennye_integraly_primery_reshenij_clip_image069.gif

Пример нахождения неопределенного интеграла

 http://www.mathprofi.ru/f/integraly_primery_reshenij_clip_image099.gif

Решение:

http://www.mathprofi.ru/f/integraly_primery_reshenij_clip_image105.gif 

(1) Используем старую- добрую формулу квадрата суммы http://www.mathprofi.ru/f/integraly_primery_reshenij_clip_image107.gif, избавляясь от степени. (2) Вносим http://www.mathprofi.ru/f/integraly_primery_reshenij_clip_image109.gif в скобку, избавляясь от произведения.

(3) Используем свойства линейности интеграла (оба правила сразу). (4) Превращаем интегралы по табличной формуле http://www.mathprofi.ru/f/integraly_primery_reshenij_clip_image073_0000.gif.

(5) Упрощаем ответ. Здесь следует обратить внимание на обыкновенную неправильную дробь  – она несократима и в ответ входит именно в таком виде. 

Задачи для самостоятельного решения.

№2.Вычислить неопределенный и определенный интеграл.

1)                                                                7)

2)                                                   8)

3)                                                                9)

4)                                                              10)

5)                                                                11)

6)                                                              12)

ДПР Р.4.3

Теоретические вопросы.

1)Первообразная функция.

2)Неопределенный интеграл.

3)Свойства неопределенного интеграла.

4)Методы интегрирования.

5)Определенный интеграл.

6)Несобственный интеграл.

7)Интегрирование.

8)Экстремумы функции.

9)Интеграл.

10)Точки экстремума.

11)Промежутки монотонности.

12)Промежутки выпуклости.

13)Асимптоты.



Предварительный просмотр:

МИНИСТЕРСТВО ОБРАЗОВАНИЯ И НАУКИ РФ.

Федеральное агентство по образованию Государственное образовательное учреждение профессионального образования КАЗАНСКИЙСТРОИТЕЛЬНЫЙ КОЛЛДЕЖ.

 ДОМАШНИЕ ПРАКТИЧЕСКИЕ РАБОТЫ.

II КУРС

Тема

 « Теория функции нескольких переменных»

Составитель преподаватель

 математики Садыкова Р.Н.

КАЗАНЬ

ДПР Р.5.1

Обучающая часть работы.

Пример: Найти частные производные: z=tg3 (3x-4y)

Находим частную производную по x , считая y константой

Находим частную производную по y , считая константой x:

Задачи для самостоятельного  выполнения .

№1.Найти.

1)z=                                               7)z=

2)z=                                        8)

3)z=                 9)z=

4)z=                                       10)z=

5)z=                                        11)-

6) z=                 12)z=

ДПР Р.5.2

Обучающая часть работы.

Найти частные производные второго порядка :

Z=

Решение:

Сначала найдем частные производные первого порядка.

Находим частную производную по x , считая y константой:

Находим частную производную по y , считая константой x:

Теперь вычисляем вторые частные производные:

Задачи для самостоятельного выполнения

1) z=                    7)   z=x5y3+

2) z=                                                        8)z=x3+xy2-3

3) z=cos)+                                                  9) z=tg(                                     

4) z=                                               10)     z=x2-2xy-4y3-52y+3                    
5) z=
                   11)  z=                                           

6) z=                                                      12) z=                                       

ДПР Р.5.3

Теоретические вопросы.

1)Функция двух переменных.

2)график функции двух переменных.

3)Окрестность.

4)Бесконечно малая.

5)Непрерывная точка.

6)Область.

7)Граничная точка области.

8)Граница.

9)Замкнутая область.

10)Ограниченная.

11)Неограниченная.

12)Область определения.

13)функция независимых переменных.

14)Дифференцирование.

15)Предел.

16)Точка разрыва.

17)частная производная.



Предварительный просмотр:

МИНИСТЕРСТВО ОБРАЗОВАНИЯ И НАУКИ РФ.
Федеральное агентство по образованию
Государственное образовательное учреждение профессионального образования
КАЗАНСКИЙСТРОИТЕЛЬНЫЙ КОЛЛДЕЖ.


ДОМАШНИЕ ПРАКТИЧЕСКИЕ РАБОТЫ.

II КУРС


Тема
« Теория обыкновенных дифференциальных уравнений»

Составитель преподаватель
математики Садыкова Р.Н.

КАЗАНЬ

ДПР Р.6.1

Обучающая часть работы.

Найти частное решение дифференциального уравнения, удовлетворяющее начальным условиям

Пример №1:

 (1 + x2) dy − 2xy dx = 0. Найти частное решение, удовлетворяющее начальному условию y(0) = 1.

 Решение:

1.Разделим переменные в уравнении. (1 + x2 )dy = 2xydx,

2.разделим на y(1 + x2), получим =

3.проинтегрируем уравнение +C

ln |y| = ln |1 + x2 | + ln |C|,

4.откуда получаем общее решение

y = C(1 + x2 ).

Чтобы найти частное решение, определим значение C по начальным условиям

 1 = C(1 + 0), C = 1.

 Следовательно, частное решение имеет вид  y(x)=1+ x2


Задачи для самостоятельного выполнения.

№ 1. Найдите общее решение дифференциальный уравнений.

1) 2

2)

3)

4)

5)

№2. Найти частное решение дифференциального уравнения, удовлетворяющее начальным условиям

 ,  ,

2) ,  , ,

3) , ,

4) ,  ,

5) ,  ,

ДПР Р.6.2

Обучающая часть работы.

Найти частное решение дифференциального уравнения, удовлетворяющее начальным условиям

Пример №1:

 (1 + x2) dy − 2xy dx = 0. Найти частное решение, удовлетворяющее начальному условию y(0) = 1.

 Решение:

1.Разделим переменные в уравнении. (1 + x2 )dy = 2xydx,

2.разделим на y(1 + x2), получим =

3.проинтегрируем уравнение +C

ln |y| = ln |1 + x2 | + ln |C|,

4.откуда получаем общее решение

y = C(1 + x2 ).

Чтобы найти частное решение, определим значение C по начальным условиям

 1 = C(1 + 0), C = 1.

 Следовательно, частное решение имеет вид  y(x)=1+ x2


Задачи для самостоятельного выполнения.

№ 1. Найдите общее решение дифференциальный уравнений.

1)

2)

3)

4)

5)

№2. Найти частное решение дифференциального уравнения, удовлетворяющее начальным условиям

 ,  ,

2) ,  , ,

3)+3 , ,

4) ,  ,

5) ,  ,


ДПР Р.6.3

Теоретические вопросы.

1) Дифференциальное уравнение.

2) Дифференциальное уравнение 1-ого порядка.

3) Порядок дифференциального уравнения.

4) Решение дифференциального уравнения.

5) Однородное уравнение.

6) Общее решение дифференциального уравнения 1-ого порядка.

7) Неоднородное уравнение.

8) Разностная сетка.




Предварительный просмотр:

МИНИСТЕРСТВО ОБРАЗОВАНИЯ И НАУКИ РФ.
Федеральное агентство по образованию
Государственное образовательное учреждение профессионального образования
КАЗАНСКИЙСТРОИТЕЛЬНЫЙ КОЛЛДЕЖ.


ПРАКТИЧЕСКИЕ РАБОТЫ.

II КУРС


Тема
« Теория вероятности»

Составитель преподаватель
математики Садыкова Р.Н.

КАЗАНЬ-2017

Практическая работа № 1

Обучающая часть работы.

          1)  Из 10 изделий, среди которых 4 бракованные, извлекают 3. Найти вероятность того, что среди них одно бракованное.

          2) Количество пятизначных чисел, составленных из пяти цифр 1,2,3,4,5, и не содержащих одинаковых цифр равно…

          3)  Количество способов расставить 9 книг на полке равно…                                                          

          4) Бросают кубик. Найти вероятность того, что выпадает число очков не меньше 3.

          5)  Сколько существует четырехзначных чисел (возможно, начинающихся с нуля), сумма цифр которых равна 5                            

          6) Известны вероятности независимых событий А, В и С: Р (А) = 0,5; Р (В) = 0,4; Р (С) = 0,6. Определить вероятность того, что а) произойдет по крайней мере одно из этих событий, б) произойдет не более 2 событий.


Задачи для самостоятельного решения.

1) Вероятности попадания в цель: первого стрелка - 0,6; второго - 0,7; третьего - 0,8. Найти вероятность хотя бы одного попадания в цель при одновременном выстреле всех трех.

2) Известно, что 80% продукции стандартно. Упрощенный контроль признает годной стандартную продукцию с вероятностью 0,9 и нестандартную с вероятностью 0,3. Найти вероятность того, что признанное годным изделие - стандартно.

3) Имеется 4 радиолокатора. Вероятность обнаружить цель для первого - 0,86; для второго - 0,9; для третьего - 0,92; для четвертого - 0,95. Включен один из них. Какова вероятность обнаружить цель?

4) В урне 4 белых и 6 черных шаров. Из урны наудачу извлечены 2 шара. Найти вероятность того, что они разного цвета

5) В лотерее из 1000 билетов имеются 200 выигрышных. Вынимают наугад один билет. Чему равна вероятность того, что этот билет выигрышный?

6) Порядок выступления 7 участников конкурса определяется жребием. Сколько различных вариантов жеребьевки при этом возможно?

Практическая работа № 2

Обучающая часть работы.

1) Известна вероятность события А: р (А) = 0,3. Дискретная случайная величина x - число появлений А в трех опытах. Построить ряд распределения случайной величины x; найти ее математическое ожидание mx и дисперсию Dx.

2) При перевозке 114 деталей, из которых 15 забракованы, утеряна 1 стандартная деталь. Найти вероятность того, что наудачу извлечённая деталь окажется стандартной.

3) На один ряд, состоящий из 18 мест, садятся 18 учеников. Найти вероятность того, что 3 определённых ученика окажутся рядом.

4) Из урны, содержащей 24 белых и 26 чёрных шаров, вынимают два шара.

5) Имеются две урны. В 1-ой лежат 19 белых и 24 чёрных шаров, во 2-ой находятся 24 белых и 21 чёрный шар. Из 1-ой урны во 2-ую перекладывают 1 шар. Какова вероятность после этого вынуть: а) белый шар из 1-ой урны; б) белый шар из 2-ой урны.

Задачи для самостоятельного решения.

1) Два производственных участка по выпуску однотипной продукции за смену выдали одинаковое количество изделий. Возможный процент брака на первом участке составляет 5%, на втором – 4%. Найти вероятность того, что наудачу взятая деталь, из числа поступивших на склад, не соответствует установленным требованиям.

2) В чемпионате по гимнастике участвуют 50 спортсменок: 17 из России, 22 из США, остальные — из Китая. Порядок, в котором выступают гимнастки, определяется жребием. Найдите вероятность того, что спортсменка, выступающая первой, окажется из Китая.

3) Производительности трех станков, обрабатывающих одинаковые детали, относятся как 1:3:6. Из не рассортированной партии обработанных деталей взяты наудачу две. Какова вероятность того, что: а) одна из них обработана на 3-м станке; б) обе обработаны на одном станке?

4) В ящике из 18 деталей 7 бракованных. Наугад выбирают две детали. Какова вероятность того, что они окажутся бракованными?

5) Если по каждой номинации установлены одинаковые призы, то порядок фильмов в комбинации 5 призов значения не имеет, и число вариантов представляет собой число сочетаний с повторениями из 10 элементов по 5, определяемое по формуле.

Практическая работа № 3

Теоретические вопросы.

1) Теория вероятности.

2) Достоверное событие.

3) Сумма двух событий.

4) Комбинаторика.

5) Независимое событие.

6) Перестановка.

7) Произведение двух событий.

8) Вероятность события.

Обучающая часть работы.

1) На 1-ом складе имеется 24 изделия, из которых 3 бракованы; на 2-ом складе находятся 29 изделий, из которых 5 бракованы. Из каждого склада выбирается по одному изделию случайным образом. После чего из этой пары отбирается одно изделие, которое оказалось небракованным. Какова вероятность, что это изделие из 1-го склада?

2) Фабрика выпускает сумки. В среднем на 130 качественных сумок приходится пять сумок со скрытыми дефектами. Найдите вероятность того, что купленная сумка окажется качественной. Результат округлите до сотых.

3) Две фабрики выпускают одинаковые стекла для автомобильных фар. Первая фабрика выпускает 45hello_html_3c6eae01.pngэтих стекол, вторая — 55hello_html_3c6eae01.png. Первая фабрика выпускает 1hello_html_3c6eae01.png бракованных стекол, а вторая — 5hello_html_3c6eae01.png. Найдите вероятность того, что случайно купленное в магазине стекло окажется бракованным.

4) В аэропорте два одинаковых автомата продают кофе. Вероятность того, что к концу дня в автомате закончится кофе, равна 0,4. Вероятность того, что кофе закончится в обоих автоматах, равна 0,2. Найдите вероятность того, что к концу дня кофе останется в обоих автоматах.

5) В случайном эксперименте бросают три игральные кости. Найдите вероятность того, что в сумме выпадет 16 очков. Результат округлите до сотых. 

Задачи для самостоятельного решения.

1) В среднем из 2000 садовых насосов, поступивших в продажу, 14 подтекают. Найдите вероятность того, что один случайно выбранный для контроля насос не подтекает.

2) Фабрика выпускает сумки. В среднем на 120 качественных сумок приходится девять сумок со скрытыми дефектами. Найдите вероятность того, что купленная сумка окажется качественной. Результат округлите до сотых.

3) В коробке лежат 5 красных, 7 зеленых и 2 синих кубика. Случайным образом из коробки берут кубик. Какова вероятность того, что из коробки взяли зеленый кубик?

4) В кармане у Сережи находится 7 монет достоинством 5 рублей, 10 монет достоинством 1 рубль и 8 монет достоинством 2 рубля. Мальчик случайным образом вытаскивает одну монету из кармана. Какова вероятность того, что будет вытащена не однорублёвая монета?

5) В чемпионате по гимнастике участвуют 50 спортсменок: 17 из России, 22 из США, остальные — из Китая. Порядок, в котором выступают гимнастки, определяется жребием. Найдите вероятность того, что спортсменка, выступающая первой, окажется из Китая.